Download as pdf or txt
Download as pdf or txt
You are on page 1of 36

LEGALEDGE TEST SERIES

MOCK COMMON LAW ADMISSION TEST 2024-25


MOCK CLAT 30
TR ID.

(In Figures)

INSTRUCTIONS TO CANDIDATES

Duration of Test : 2 Hours (120 Minutes)

c o m Maximum Marks : 120


1. Separate carbonised Optical Mark Reader
(OMR) Response Sheet is supplied along with
this Questions Booklet and the carbon copy
10.

rs .
Use BLACK/BLUE BALL POINT PEN only for
writing the roll No. and other details on OMR
response Sheet.
has to be detached and taken by the
candidates.

k e11. Use BLACK/BLUE BALL POINT PEN for


shading the circles. Indicate only the most

n
2. In case of any discrepancy in the question appropriate answer by shading from the
booklet (QB), please request the invigilator for options provided. The answer circle should be

for a fresh booklet so obtained.

p r a
replacement of a fresh packet of QB with OMR.
Do not use the previous OMR response Sheet 12.
shaded completely without leaving any space.
As the responses cannot be modified/corrected
on the OMR Response Sheet, candidates have

o
7F3A 0B9C7E1D
3. Candidatestr-5E will not be given a second blank to take necessary precautions before marking

T
OMR response Sheet under any circumstance. the appropriate circle.
Hence, OMR response Sheet shall be handled 13. The candidate should retain the Admit Card
carefully. duly Signed by the invigilator, as the same has
4. Answer all questions. No clarification can be to be produced at the time of Admission.
sought on the Questions Paper 14. Handle the OMR response Sheet with care. Do
5. Possession of electronic devices in any form is not fold.
strictly prohibited in the examination Hall. 15. Ensure that invigilator puts his/her signature in
6. The use of any unfair means by any candidate the space provided on the OMR response
shall result in the cancellation of his/her tr-5Q7P3O0S9L
7U1M Sheet. Candidate should sign in the space
examination. provided on the OMR response Sheet.
7. Impersonation is an offense and the candidate, 16. The candidate should write Question Paper
apart from disqualification, will be liable to be booklet No., and OMR response Sheet No.,
prosecuted. and sign in the space/column provided in the
8. The test Paper for Five Year integrated Law attendance sheet.
Programme is for 120 marks containing 120 17. Return the Original Page of OMR response
9L7U1M
5Q7P3O0SChoice
tr-multiple Questions. Sheet to the invigilator after the examination.
9. There will be Negative marking for multiple 18. The candidate shall not write anything on the
choice objective type questions. 0.25 marks OMR response Sheet other than the details
will be deducted for every wrong answer or required and in the spaces provided for.
where candidates have marked more than one
response.
CONTENT OF QUESTION PAPER

Subject Q. No. Page No.


English Language 01-24 3
Current Affairs and General Knowledge 25-52 10
Legal Reasoning 53-84 17
Logical Reasoning 85-108 29
Quantitative Techniques 109-120 34

c o m
rs .
k e
r an
tr-5E7F3A0B9C

o
7E 1D

p
T
7U1M
tr-5Q7P3O0S9L

7U1M
tr-5Q7P3O0S9L

Head Office: 127, Zone II, MP Nagar, Bhopal |+91-7676564400| https://www.toprankers.com Page 2 of 36
SECTION-A : ENGLISH LANGUAGE

Directions (Q.1-Q.24): Read the passage carefully and answer the questions.
Passage (Q.1-Q.6): Culture is an agglomeration of human labour. It is made up of the social, political, religious,
and literary ethos of humans. Culture is the building block of human society. It is a continuous variable. Society
is a result of human behavior under different circumstances and it also defines its culture. As culture is an
evolving process, there is a conflict between the human conscience and the circumstances of the moment. So
culture can be defined as the result of this conflict. It is a refinement of every aspect of human nature.
The development of Indian society is deeply rooted in its various cultures. Its different cultures acted as
tributaries to become a vast, time-tested and inseparable entity that we know today as Indian culture or, in other
words, cultures of India. Its core strength lies in its interdependency because it is a multicultural society. As our

m
national song says, from the Himalayas to the Indian Ocean, the country presents a carnival of different social,
cultural, and geographical diversities. That’s why diversity and coexistence are so important to culture, which is

.c o
now wrongly depicted by some groups as a unilateral religious thought process. On the contrary, the cultures of
India teach us about vasudaiva kudumbakam (the world is one family). Culture, then, is a result of argumentative

rs
and dialectical conversations on different ideologies and philosophies.

e
The Vedas, the Upanishads, the Ramayana, the Mahabharata, Sangam literature have ignited dreams, stories,
myth, history, fantasies, spiritualities and ideologies of Indian society for many centuries. On the one hand, there

k
is Adi Sankara who taught us the principles of Advaita and, on the other, we have Sree Narayana Guru and

n
Chattambiswamy who fought against blind faith. This coexistence of harmony and conflict can also be found in

a
our classics and folklore. Tulsidas and Thunchaththu Ramanujan Ezhuthachan, who were scholars of Sanskrit

p r
language, created their masterpieces — Ramcharitamanas and Adhyathmaramayanam Kilippattu in the
languages of the masses. In this context, I remember the words of noted Hindi writer Nirmal Verma: “Only in

o
9C7E1D
5E7F3A0Band
the time oftr-labour creation, man builds himself. In other words, the nature of his culture lies within the
language of his labour.”

T
We see different ideological layers presenting Lord Rama in different perspectives by Valmiki, Tulsidas and
Naresh Mehta. Tulsidas began his work, moved by the death of a male krauncha bird at the hands of a hunter
and the piteous cries of the female on the loss of its mate. Sorrow and compassion had moved him.
The cultures of India, in essence, is about socialism which gives equal importance to man and nature. It teaches
us about collectiveness, harmony, welfare of all, tolerance, coexistence and, last but not the least, about love.
Source: Extracted with edits and revisions from the article, “The many cultures of India”, written by Mr.
1M
Pradeep Raj and published in The Hindu. tr-5Q7P3O0S9L7U
1. What is the connection between the first and second paragraphs of the passage?
(a) The first paragraph defines culture as a product of ongoing human interactions and conflicts, while the second
paragraph illustrates these concepts through the example of Indian society's cultural amalgamation.
(b) The first paragraph outlines the theoretical aspects of culture as an evolutionary process, whereas the second
paragraph
7U1M
addresses the practical outcomes of these processes in the context of Indian society's development.
tr-5Q7P3O0S9L
(c) The first paragraph posits culture as an evolving entity formed by a conflict of conscience and circumstances,
which the second paragraph exemplifies by detailing the resultant diversity and interconnectedness of Indian
culture.
(d) The first paragraph discusses culture's dependency on human nature and social constructs, and the second
paragraph showcases this dependency through the lens of Indian culture’s reliance on multicultural
integration.

Head Office: 127, Zone II, MP Nagar, Bhopal |+91-7676564400| https://www.toprankers.com Page 3 of 36
2. The primary reason the author calls culture a refinement is that:
(a) it undergoes a systematic enhancement and improvement over time, mirroring the development of human
society and its underlying ethical values.
(b) it represents a distillation and enhancement of human attributes, emerging from the continuous interactions
and conflicts between conscience and circumstances.
(c) it functions as a mechanism for societal progress, inherently fostering greater complexity and sophistication
in social structures and norms.
(d) it encapsulates the best qualities of human civilization, constantly being polished and redefined through
ongoing societal and political developments.
3. The author would definitely agree to which of the following?
(a) Despite the pervasive challenges and conflicts inherent in its development, Indian culture fundamentally
champions unity and the collective welfare of its society.

o m
(b) The complexities and contradictions within Indian culture primarily impede the effective integration of its
diverse social elements.

c
historical and philosophical foundations.

rs .
(c) Cultural evolution in India has been predominantly driven by external influences rather than its intrinsic

(d) The primary function of culture is to maintain historical continuity rather than to adapt to new challenges or
integrate new ideas.

k e
n
4. All the following pairs of words are synonyms, except:
(a) Amalgamation – Confluence

a
(b) Dialectical - Argumentative

r
(c) Enhancement – Refinement (d) Multicultural – Monolithic
5.
(a) A textbook 3A0B
tr-5E7Fon comparative
p
What type of publication is this passage likely extracted from?

o
9C7E1D politics emphasizing different governance models around the world.
(b) A travel guidebook detailing various tourist destinations and cultural attractions in India.

society.
T
(c) An academic journal article discussing the sociocultural dynamics and historical influences on Indian

(d) A brochure for an international cultural festival showcasing performances and exhibitions from various
countries.
6. What is the main idea of the passage?
(a) The passage primarily discusses the role of literary figures in shaping the philosophical dimensions of Indian
culture. 7U1M
tr-5Q7P3O0S9L
(b) The passage explores the evolution and richness of Indian culture, emphasizing its diversity, the impact of
historical texts, and the importance of coexistence and unity.
(c) It outlines the geographical diversity of India, linking it to the variety of cultural expressions found
throughout the country.
(d) The text advocates for a reinterpretation of Indian culture through the modern lens of social and political
ideologies.
1M
7U
tr-5Q7P3O0S9L

Head Office: 127, Zone II, MP Nagar, Bhopal |+91-7676564400| https://www.toprankers.com Page 4 of 36
Passage (Q.7-Q.12): The 2024 general election distinguished itself with many unprecedented occurrences, not
least of which was the starkly ruthless and expensive digital campaigns orchestrated by political parties. This
emergent paradigm in electoral strategy illuminated a big transformation in the construction, distribution, and
reception of political messaging by the voting populace.
It is emblematic of an epoch where conventional canvassing is progressively eclipsed by digital prowess.
Consequently, the electoral arena underwent a profound reconfiguration, spurred by the substantial escalation in
the number of actively engaged social media users across India over the preceding decade. This led to a
recalibration of political engagement strategies, as parties tried to harness the vast potential of digital platforms
to cultivate voter sentiment and sway electoral outcomes.
India’s ruling Party emerged as the primary spender of Google Ads, spending Rs 116 crores within the initial
five months of the election year 2024. During the period from January to May 2024, the aggregate expenditure

m
on all political advertisements surged to Rs 288 crores, marking a tenfold escalation compared to the

o
corresponding period in the previous 2019 election cycle. YouGov Surveys indicate that in urban India, 76%

c
attest to having been exposed to political advertisements during this general election, with 14% reporting

rs .
otherwise. An examination of the state-wise expenditure on political advertisements serves as a poignant
indicator of the intensity of electoral competition within each respective state, and the intensity of the contest.
Odisha emerged as an unexpected newcomer atop the list, surpassing Andhra Pradesh, Tamil Nadu, Maharashtra,
Uttar Pradesh, and Bengal in terms of digital ad-spend.

k e
n
The conspicuous escalation of surrogate campaigning across Facebook and Instagram revealed more. Pages on

a
these two sites were often clandestinely endorsed with the aim of influencing public opinion under the guise of

r
organic support. Primarily, these social media handles disseminated defamatory content targeting political

p
adversaries, frequently resorting to ridicule through memes and animated videos.

o
While financial 0B9C7E1Dinevitably shape democratic expression, there is a compelling imperative to scrutinise
resources
tr-5E7F3A
and regulate the unethical utilisation of digital tools and social media platforms. A recent investigation by Al

T
Jazeera has unveiled that India’s foremost social media election advertisers use Facebook pages illicitly sold and
brought in contravention of the company’s own regulations. Such revelations cast a shadow over the integrity of
the electoral process, exposing the rampant proliferation of black market networks within platforms like Meta
and YouTube.
https://thewire.in/politics/election-digital-campaign-parties
7. What do you think is the source of the passage?
(a) A science fiction novel 7U1M
tr-5Q7P3O0S9L
(b) A research paper on digital communication
(c) An investigative journalism report
(d) A personal blog post
8. Identify the part of speech of the underlined word in the following sentence:
"An examination of the state-wise expenditure on political advertisements serves as a poignant indicator of the
tr-intensity of7Uelectoral
1M competition."
5Q7P3O0S9L
(a) Noun (b) Verb (c) Adjective (d) Adverb

Head Office: 127, Zone II, MP Nagar, Bhopal |+91-7676564400| https://www.toprankers.com Page 5 of 36
9. Based on the passage, how does the influence of digital campaigning on electoral integrity contrast with the
handling of these platforms by regulatory bodies?
(a) The passage suggests that digital campaigns, while expanding reach, compromise electoral integrity due to
unethical practices, highlighting a gap in effective oversight by regulatory bodies.
(b) Digital campaigning enhances electoral integrity by increasing transparency and voter information, unlike
regulatory bodies which struggle to keep up with technological advances.
(c) Regulatory bodies are shown to be proactive and stringent, effectively countering the manipulative tactics
used in digital campaigns.
(d) Both digital campaigns and regulatory bodies are depicted as fully aligned in their objectives to enhance
democratic participation without any conflicts or issues.
10. Analyze the use of the term "surrogate campaigning" as mentioned in the passage. What does its use imply about

m
the nature of digital electoral strategies?
(a) Surrogate campaigning signifies a strategic, yet undisclosed collaboration between digital platforms and

.c o
political campaigns to enhance message reach, often appearing as spontaneous supporter activities.
(b) It suggests that digital campaigns are directly managed and executed by the official campaign teams without

rs
third-party involvement.
(c) Surrogate campaigning refers to the indirect or concealed support for political entities under the guise of
organic social media movements, implying a level of deception.

k e
(d) This term describes a fully legal and ethical practice where surrogate campaigners openly advocate for their

n
preferred candidates without using digital platforms.
11.

outcomes."

p r a
Identify the phrasal verb in the following sentence: "This led to a recalibration of political engagement strategies,
as parties tried to harness the vast potential of digital platforms to cultivate voter sentiment and sway electoral

o
1D
(a) Led to tr-5E7F3A0B9C7E(b) Tried to (c) Cultivate voter (d) Sway electoral
12.

T
Identify the antonym of the word 'eclipsed' as used in the passage: "It is emblematic of an epoch where
conventional canvassing is progressively eclipsed by digital prowess."
(a) Obscured (b) Highlighted (c) Diminished (d) Shadowed
Passage (Q.13-Q.18): The ‘human condition’ has attracted increasing interest in management studies. Since a
large part of our lives is spent at work and in other organizational contexts, it only makes sense to attend to the
experiences, perceptions, and struggles of organizational members. Employees may have been treated as
anonymous and interchangeable ‘human capital’ in the past, but the daily lives of individual members are
currently gaining increasing attention. Often,tr-there
5Q7P3O 9L7Ua1Mpredilection for studying managers and executives,
is0Sstill
but there is a growing interest in lower-level personnel too. They feature prominently in research on empathy,
violence, alienation, gendered practices, disruptions, domination, and inter-personal relations in organizations.
Indeed, the information that is required for studying the human condition in organization is deeply personal. It
requires researchers to delve into the inner lives of organizational members and explore intimate questions. It is
therefore no 1Msurprise that scholars resort to qualitative methods for data collection. But still, people might not
5Q7P3O0S9L7U
tr-readily share this private information. Similarly, observations in organizations may offer some insight into
practices of resistance and enactments of identity, but do not tell us how members give meaning to these actions
themselves, nor provide insight into their thoughts and feelings. Fictional narratives have emerged as an
alternative source of data.
Fiction, refers to a story in which characters, plot, and events are invented by the author(s). Examples include
novels, etc. A fictional story can be loosely based on the author’s own life or largely imagined, but readers are
generally able to empathize with the experiences of the main characters. Indeed, fiction offers us a rare insight
Head Office: 127, Zone II, MP Nagar, Bhopal |+91-7676564400| https://www.toprankers.com Page 6 of 36
into what it means to be human and, in many cases, what it means to be an organizational member. Even though
fictitious, such stories demonstrate the lived experience of work or organizational membership that will be
familiar to most of us .They show inter-personal relations and conflicts that are similar to the ones that we have
all experienced or witnessed. And they highlight emotions and doubts that we confront on a daily basis, but find
hard to put into words. As such, fiction contributes to our knowledge on the inner lives of the people who inhabit
organizations.
Fiction studies have facilitated by the fact that the linkages between fiction and organizational analysis are
manifold. For one, there is the belief that any organizational text is to some extent fictional. Executives come up
with narratives to justify their decisions to stakeholders, managers invent stories that will help them convince
their subordinates. In turn, scholarly texts in our field do not present objective truths, but are socially constructed
artifacts that fit certain paradigms. This realization led scholars to reject the sharp distinction between science
and fiction.

13.
https://journals.sagepub.com/doi/full/10.1177/16094069241229781

o m
Which of the following best describes the context in which this passage would most likely appear?

c
.
(a) A scholarly article analyzing the psychological impacts of fiction in understanding workplace behaviors.

rs
(b) A research paper examining the interplay between fiction and reality in the study of organizational behavior.
(c) A textbook on human resource management that integrates fictional narratives to illustrate key concepts.

14. How frequently is the main idea emphasized in the passage?

k e
(d) An academic journal discussing the use of fiction as a research method in organizational studies.

n
(a) The main idea is touched upon briefly at the beginning and not mentioned again.

a
(b) The main idea is a recurring theme discussed throughout the passage.

r
(c) The main idea is only implied in the conclusion of the passage.

15.
tr-5E7F3A0B9C

o
7E 1D

p
(d) The main idea appears sporadically without much emphasis.
Arrange the following sentences to form a coherent paragraph that reflects the main themes of the passage.

T
1) Fictional narratives provide a unique perspective into the human condition, particularly within organizational
settings, showcasing interpersonal relations and personal conflicts familiar to many.
2) Observations and qualitative research delve into the personal aspects of organizational life, yet often fall
short of capturing the subjective experiences and meanings attributed by organizational members themselves.
3) The human condition has garnered considerable attention in management studies, emphasizing the
significance of attending to the personal and daily experiences of all organizational members.
4) Fiction, as an alternative source of data, is valued for its ability to convey the nuanced emotions and situations
that organizational texts, often perceived as partially fictional,
7U 1M struggle to express.
tr-5Q7P3O0S9L
5) Increasing focus on lower-level personnel in studies on empathy, violence, and alienation marks a shift from
traditional research predominantly centered on managers and executives.
(a) 3, 5, 2, 1, 4 (b) 2, 4, 1, 3, 5 (c) 5, 3, 1, 4, 2 (d) 4, 2, 3, 5, 1
16. Identify the part of speech of the underlined word in the following sentence:
"Executives come up with narratives to justify their decisions to stakeholders."
tr-(a)
5Q7PNoun
3O0S9L7U1M (b) Verb (c) Adjective (d) Adverb

Head Office: 127, Zone II, MP Nagar, Bhopal |+91-7676564400| https://www.toprankers.com Page 7 of 36
17. Which of the following best represents the conclusion of the passage?
(a) Fiction serves as a critical tool in revealing the nuanced emotional landscape and interpersonal dynamics
within organizations, often missed by other research methodologies.
(b) The personal experiences and emotional states of organizational members are now considered vital areas of
study in management.
(c) The effectiveness of traditional research methods in capturing the complete human experience within
organizations is being reevaluated.
(d) The blending of fiction and traditional research methods offers a more holistic understanding of
organizational behavior.
18. In the provided excerpt, what is the synonym for "predilection" as used in the context of organizational studies?
(a) Indifference (b) Preference (c) Aversion (d) Ignorance

o m
Passage (Q.19-Q.24): Alcohol use and misuse have been part of human society for centuries. Early physicians
recognized since the 1800s that excessive and daily alcohol intake produced not only impairment of the senses

c
.
but also higher predisposition for respiratory illnesses. William Osler, the father of scientific medicine, reported

rs
in 1905 that patients who misused alcohol had higher predisposition to pneumonia. The rationale for this
predisposition for respiratory illnesses is mentioned at the far end of this written piece.

k e
Between 2006 and 2010 in the United States, excessive alcohol consumption resulted in approximately 88,000
deaths (data based on 11 U.S. States), and the median alcohol-attributable death rate was 28.5 per 100,000

n
population. Furthermore, the potential years of life lost attributed to alcohol (estimate of the average years people

a
would have lived if they had not died prematurely) averaged 2.5 million years annually from 2006 through 2010.

r
Importantly, the majority of alcohol-related deaths and potential life lost were among working-age adults (20-

p
to 64-year-olds). In addition, the estimated cost of excessive drinking was $223.5 billion in 2006, from which

o
the majority represented
tr-5E7F3A0B9C
loss of productivity, valued at $161.3 billion, followed by increased health care costs
7E1D
and criminal justice costs of $21 billion each. In India, the picture is bleaker. The epitome of formal studies on

T
relation between alcohol and crime is the comprehensive statistics report published by the National Crime
Records Bureau (‘NCRB’, for brevity), Ministry of Home Affairs. In its latest avatar released in starting of this
momentous year ‘2020’, the incidence volume of crime related to ‘Liquor and Narcotic Drugs’ came out
at 8,63,696, accounting for 65.3 % of the crime rate.
Alcohol misuse has excessive public health and criminal justice costs. Alcoholic myopathy is a condition
involving muscle weakness and loss of muscle due to abnormal breakdown of muscle tissue. This muscular
degeneration leads to muscle dysfunction, which impacts 1M various parts of the body and their functionality and
5Q7P3O0S9L7U
can be either acute or chronic. For more tr-than a century, physicians have noticed that alcohol produces
abnormalities in the host defenses. As early as the late 1800s, scientists performed studies to determine how
ethanol produced detrimental effects in humans, despite not being toxic. This does not include the extremely
poisonous effects of illegally-distilled methanol. Human studies have provided important epidemiological data
demonstrating an association between alcohol consumption and risk of infection. This increased risk of infection
has been attributed to alcohol’s effect on the immune system. The intestinal mucosa plays an important role in
tr-alcohol 9L7U1M
5Q7P3O0Smetabolism, as the epithelial surface incorporates alcohol into the blood system by passive diffusion,
which accounts for approximately 80 percent of alcohol absorption. The other 20 percent is absorbed through
the gastric mucosa. Alcohol induces expression of the enzyme CYP2E1, increasing reactive oxygen species,
which damage tissue through increases in oxidative stress and compromised immune system. Alcohol also
increases the expression of circadian clock genes that alter intestinal permeability.
[Source: https://www.ncbi.nlm.nih.gov/pmc/articles/PMC4590615/]

Head Office: 127, Zone II, MP Nagar, Bhopal |+91-7676564400| https://www.toprankers.com Page 8 of 36
19. What is the central thesis in the extract given above?
(a) Alcohol misuse has been linked to many aspects of weak muscles while having little effects on criminal
activities.
(b) Alcohol has been linked with higher crime rate despite being of little significance in determining health.
(c) There are health and criminal repercussions to the misuse of alcohol in the two countries whose data is
presented.
(d) United States and India have been bearing the brunt of health and criminal aspects related to alcohol as
medicine.
20. Which of these cannot be inferred from the passage?
(a) There is prevalence of respiratory illnesses in person who consume alcohol in diminutive amounts
sporadically.

m
(b) All kinds of alcohol are not to be considered edible.

o
(c) Majority of the crimes in the total percentage in India happen due to alcohol and drugs.

c
(d) Alcohol is responsible for majority of demise in working age adults in United States.
21.

rs .
What is the reason that alcohol consumption is responsible for increase in respiratory illnesses in accordance
with the passage?

e
(a) Because it increased oxidative stress and compromises immune system.
(b) Because it has been linked to promoting the proliferation of bacteria and virus.

n
(d) Because it increased alkaline stress and compromised immune system.

a k
(c) Because it has been linked to increasing toxicity in the system of the human body.

22.

r
What is the percentage of the crime rate which would be unaffected if there is total ban on alcohol and drugs in
India?

p
o
(a) Approximately thirty
tr-5E7F3A0B9C
7E1Dfive percent. (b) Approximately sixty five percent.
(c) Approximately thirty three percent. (d) Approximately sixty seven percent.
23.

24.
T
What is the scientific name of the alcohol mentioned in the passage which produces an extreme toxicity?
(a) Ethanol. (b) Methanol. (c) Propanol (d) Iso-Propanol.
Which of these is not a disease which is caused or aggravated by the misuse of alcohol as per the passage?
(a) Muscular myopathy. (b) Pneumonia.
(c) Respiratory illnesses. (d) Muscular kryopathy.

7U1M
tr-5Q7P3O0S9L

7U1M
tr-5Q7P3O0S9L

Head Office: 127, Zone II, MP Nagar, Bhopal |+91-7676564400| https://www.toprankers.com Page 9 of 36
SECTION -B : CURRENT AFFAIRS, INCLUDING GENERAL KNOWLEDGE
Directions (Q.25-Q.52): Read the information carefully and answer the questions.
Passage (Q.25-Q.30): UNSC Reforms and UNECOSOC
India has presented a detailed model on behalf of the G4 nations for Security Council reform that includes new
permanent members elected democratically by the General Assembly and displays flexibility on the veto issue.
Participating in the Inter-governmental Negotiations on Security Council reform (IGN) on March 7, India’s
Permanent Representative to the UN Ambassador Ruchira Kamboj said the UN’s 80th anniversary next year
serves as a milestone to achieve concrete progress on the long-pending subject.
Ms. Kamboj presented the ‘G4 model’ on behalf of Brazil, Germany, Japan and India for debate, dialogue and
finally negotiations. The proposals elicited strong support from wider UN members.

o m
“The realities of 1945, when the Council was established, have long been superseded by the geo-political realities
of the modern era and a new century; with the need for change being felt across the board,” Ms. Kamboj said as

c
she shared the exhaustive G4 model with UN Member States in the General Assembly.
Extracted with edits and revisions from https://www.thehindu.com

rs .
e
25. The Security Council was established in response to the failures of the League of Nations in maintaining world
peace. In which year was the Security Council formed, as replaced by [1] in the above passage?

26.
(a) 1939 (b) 1942 (c) 1945

an
(d) 1948

k
Consider the following statements about the United Nations Security Council and choose the one that is TRUE?

r
(a) The United Nations Security Council consists of 25 permanent members.

p
(b) The headquarters of the United Nations Security Council is in Geneva, Switzerland.
(c) India is a permanent member of the United Nations Security Council.

o
7E1D
tr-5E7F3A0B9C
(d) The United Nations Security Council has five permanent members, each with veto power.
27.

T
He was awarded the Nobel Peace Prize posthumously for his efforts in resolving global crises and remains the
only individual to receive this honour after his death. His diplomatic skills and administrative competence during
his tenure at the United Nations were highly acclaimed. What was the name of this esteemed diplomat and
administrator?
(a) Trygve Lie (b) Kurt Waldheim
(c) Ban Ki-moon (d) Dag Hammarskjöld
28. Which of the following best describes the membership and role of the UN Economic and Social Council
7U1M
tr-5Q7P3O0S9L
(ECOSOC)?
(a) ECOSOC has 54 members elected by the UN General Assembly for overlapping three-year terms,
coordinating economic, social, and environmental issues.
(b) ECOSOC has 54 members elected for five-year terms, focusing exclusively on global health initiatives.
(c) ECOSOC includes representatives from all UN member states and primarily deals with military conflict
resolution.
7U1M
tr-5Q7P3O0S9L
(d) ECOSOC operates under the direct control of the UN Secretary-General and manages only environmental
sustainability programs.
29. Recently, the UN Secretary-General recently invoked Article 99 of the UN Charter in response to Israel's actions
in Gaza. Which of the following statements is NOT TRUE about Article 99 of the UN Charter?
(a) Article 99 empowers the Secretary-General to independently bring any matter they believe may threaten
international peace and security to the attention of the Security Council.
(b) Article 99 has been frequently invoked by Secretary-Generals to manage international crises.
Head Office: 127, Zone II, MP Nagar, Bhopal |+91-7676564400| https://www.toprankers.com Page 10 of 36
(c) Article 99 is considered discretionary, allowing the Secretary-General to highlight critical issues that require
the Security Council's attention.
(d) Previous invocations of Article 99 include addressing issues in the Republic of the Congo in 1960 and the
creation of Bangladesh in 1971.
30. What is the required process for amending the Charter of the United Nations, particularly concerning reforms to
the Security Council?
(a) A simple majority in the General Assembly followed by ratification by all member states.
(b) A two-thirds majority in the General Assembly, then ratification by two-thirds of the member states,
including all permanent Security Council members.
(c) Unanimous approval by the Security Council followed by ratification by the General Assembly.
(d) Approval by the International Court of Justice followed by a global referendum among all UN member states.
Passage (Q.31-Q.36): Collegium System

o m
In a six-page order, Supreme Court Registrar Puneet Sehgal explained that the Collegium system had already

c
been upheld, while the NJAC — which gave an equal role to the government in judicial appointments — had

subsequently dismissed in 2018.

rs .
been struck down by a Constitution Bench in October 2015. A review plea against the judgment was also

k e
The petition filed by advocate Mathews Nedumpara only “replicated” issues which have already been put to rest,
Mr. Sehgal said. “It is critical to ensure litigants do not overburden courts with matters which already stand
adjudicated,” he observed in the order dated April 24.

an
A repeat litigation was not in the public’s best interest, the order said, adding that the petition by Mr. Nedumpara

r
was either an attempt to overreach the principles of settled law or fuelled by an “ulterior motive”.
The Registrar invoked
tr-5E7F3A0B9C
7Ethe

o p
1D Supreme Court Rules, 2013 in his order. “I hold that the registration of the present
case was not proper and by virtue of Order XV Rule 5 of the Supreme Court Rules, 2013, I hereby decline to

T
receive the same,” the order declared.
Under this provision of the 2013 Rules, the Registrar may refuse to receive a petition on the ground that it
discloses no reasonable cause or is frivolous or contains scandalous matter. The petitioner has 15 days to appeal
to the court.
Extracted with edits and revisions from https://www.thehindu.com
31. Which constitutional amendment, aimed at establishing the National Judicial Appointment Commission (NJAC),
7U1M
tr-5Q7P3O0S9L
was struck down by the Supreme Court of India?
(a) The Constitution (Forty-second Amendment) Act, 1976
(b) The Constitution (Forty-fourth Amendment) Act, 1978
(c) The Constitution (Ninety-ninth Amendment) Act, 2014
(d) The Constitution (One Hundred First Amendment) Act, 2016
32. What is the7Uestablished
1M procedure for appointing the Chief Justice of India (CJI)?
tr-5Q7P3O0S9L
(a) The President of India appoints the CJI, who is recommended by the outgoing CJI, typically based on
seniority.
(b) The outgoing CJI nominates his successor, typically the most junior Supreme Court judge, for presidential
appointment.
(c) The President of India appoints the CJI based on the recommendation of the Union Law Minister.
(d) The Prime Minister nominates the CJI, and the Parliament must confirm the appointment through a simple
majority vote.

Head Office: 127, Zone II, MP Nagar, Bhopal |+91-7676564400| https://www.toprankers.com Page 11 of 36
33. The NJAC Act was declared unconstitutional on October 16, 2015, in the case of Supreme Court Advocates-on-
Record Association v. Union of India. Which Justice of the then Supreme Court provided the sole dissenting
opinion in this case?
(a) Justice J. Chelameswar (b) Justice Madan B. Lokur
(c) Justice Kurian Joseph (d) Justice T.S. Thakur
34. How is a Supreme Court Justice appointed in the United States?
(a) The President of the United States nominates a candidate, who must then be approved by a simple majority
in both houses of Congress.
(b) The President of the United States nominates a candidate, who must then be confirmed by a majority in the
Senate.
(c) The Chief Justice of the United States nominates a candidate, who must then be confirmed by the Senate

m
Judiciary Committee.
(d) The Senate nominates a candidate, who is then appointed by the President if a majority of the House of

35.
Representatives agrees.

.c o
In which of the following years was the strength of the bench in the Supreme Court of India increased from 31

rs
to 34 justices?
(a) 2009 (b) 2014 (c) 2019 (d) 2022
36.

k e
In November 2019, the then Chief Justice of India, S.A. Bobde, launched a machine-assisted translation tool
designed to facilitate the inclusion of regional languages in judicial proceedings. This tool uses Artificial

(a) JUDIS Translate (b) Legal Lang

r n
Intelligence (AI) for its operations. What is the name of this software?

a
(c) SUVAS (d) Adalat Anuvad
Passage (Q.37-Q.42): South China Sea
Central to tr-
recent

o
5E7F3Astandoffs

p
0B9C7E1D between the Philippines and China are two hotly contested features located inside
Manila's 200-nautical mile exclusive economic zone, but which Beijing claims as its own.

T
China uses the so-called nine-dash line that takes in about 90% of the South China Sea to assert its claim to
sovereignty over the Scarborough Shoal, a submerged reef coveted for its bountiful fish stocks, and the Second
Thomas Shoal, home to a small contingent of Filipino sailors living aboard a rusty warship that Manila
intentionally grounded in 1999 to further its territorial claims. Encounters between the Philippines and China in
Asia's most contested waters have grown more tense and frequent over the past year as Beijing pressed its claims
and Manila refused to cease its fishing and resupply activities to Filipinos at the two shoals. China considers
those to be illegal intrusions and has tried to repel the vessels.
1M
5Q7P3O0S9L7U
China's coastguard has stepped up so-calledtr-"grey-zone" activities such as use of water cannon, collision and
ramming tactics, and, according to Manila, use of a military-grade laser, to try to stop the Philippine resupply
and patrol missions. It has also deployed an armada of fishing boats the Philippines and allies consider militia.
Extracted with edits and revisions from https://www.reuters.com
37. In 2016, an UNCLOS arbitral tribunal made a significant ruling regarding the territorial disputes in the South
China Sea7Ubetween
tr-5Q7P3O0S9L
1M China and the Philippines. Which of the following was a key finding of the tribunal's
decision?
(a) The tribunal affirmed China’s historical claims over the South China Sea based on the nine-dash line.
(b) The tribunal decided to allow joint development of resources by China and the Philippines in the disputed
area.
(c) The tribunal ruled that both China and the Philippines have equal rights to the Scarborough Shoal.
(d) The tribunal found that China’s nine-dash line claim had no legal basis and violated the Philippines'
sovereign rights.
Head Office: 127, Zone II, MP Nagar, Bhopal |+91-7676564400| https://www.toprankers.com Page 12 of 36
38. Which of the following countries does not border the South China Sea?
(a) Laos (b) Malaysia (c) Vietnam (d) Brunei
39. With the Act East Policy initiated in 2014, the relationship with the Philippines has diversified further into
political security; trade, industry, and people-to-people realms. Which statement correctly describes the 'Act East
Policy' of India, announced in November 2014?
(a) A military strategy focusing solely on defence collaborations with Eastern European countries.
(b) A continuation and enhancement of the "Look East Policy" focusing on the Asia-Pacific region.
(c) A policy aimed at decreasing economic ties with Southeast Asian countries.
(d) A domestic policy initiative focused on improving north-eastern India's infrastructure without international
involvement.
40. Which of the following statements accurately describes the ongoing disputes in the South China Sea?
(a) Brunei, Indonesia, and Vietnam contest the sovereignty of Scarborough Shoal.

o m
(b) China, Taiwan, and Vietnam claim sovereignty over the Paracel Islands, with China having occupied them
since 1974.

c
(d) Taiwan exclusively occupies all major land features in the Spratly Islands.

rs .
(c) The Philippines exclusively claims all of the Spratly Islands, without contest from other nations.

41.

k e
In a recent geopolitical move, the Chinese Ministry of Civil Affairs released a list of standardized geographical
names for an area they refer to as "Zangnan," which is claimed by Beijing as part of south Tibet. India has
strongly rejected this naming, affirming that the region is an integral part of its territory. What is the Indian name
for the region that China refers to as "Zangnan"?
(a) Ladakh
(c) Arunachal Pradesh

r an
(b) Sikkim
(d) Nagaland
42. Which of thetr-5Efollowing

o
7F3A0B9C7Estatements
1D

p
correctly defines the Exclusive Economic Zone (EEZ) as per the 1982 United
Nations Convention on the Law of the Sea (UNCLOS)?

T
(a) The EEZ extends up to 200 nautical miles from a country's coastline, and the country has shared rights to
the resources with neighbouring countries.
(b) The EEZ extends up to 200 nautical miles from a country's coastline, and the country has exclusive rights to
the exploration and exploitation of natural resources within this area.
(c) The EEZ extends up to 200 nautical miles from a country's coastline, and the country must share the resources
with the international community.
(d) The EEZ extends up to 200 nautical miles from a country's coastline, and the country has no special rights
to the resources within this area. 7U1M
tr-5Q7P3O0S9L

Passage (Q.43-Q.47): FDI Allowed in Space;


India will allow 100% foreign direct investment in the manufacture of satellite systems without official approval
and eased the rules for launch vehicles, a government statement said, aiming for a greater share of the global
space market.
India's space ambitions got a boost when it became the first country to land a spacecraft near the unexplored
7U1M
tr-5Q7P3O0S9L
south pole of the moon in August - and the fourth to achieve a soft landing - just days after a similar Russian
mission failed.
The government said in a statement late on Wednesday that foreign companies could invest in the manufacture
of components and systems or sub-systems for satellites up to 100% without approval.
Foreign firms planning to build satellites in India would not require government approval up to 74% of the
investment; for investment in launch vehicles, investment could go up to 49% without such approval, the
statement said.
Head Office: 127, Zone II, MP Nagar, Bhopal |+91-7676564400| https://www.toprankers.com Page 13 of 36
India has privatised space launches and is aiming for a five-fold increase in its share of the global launch market,
which some expect to be worth $47.3 billion by 2032. India currently accounts for about [1] of the space
economy. Extracted with edits and revisions from https://www.weforum.org
43. India currently accounts for about [1] of the space economy. Which of the following percentages has been
replaced with [1] in the above passage?
(a) 1% (b) 2% (c) 5% (d) 10%
44. Which of the following statements is incorrect about the Indian Space Policy 2023?
(a) The policy only allows ISRO to conduct space launches and prohibits private sector involvement.
(b) InSPACe will serve as a single window clearance and authorisation agency for space activities, including
launches and satellite transactions.

m
(c) New Space India Limited (NSIL) is responsible for commercializing space technologies and leasing or
procuring space components from both private and public sectors.

global space governance.

.c o
(d) The Department of Space will provide overall policy guidelines and coordinate international cooperation in

rs
45. In 2023-24, which country provided the highest foreign direct investment (FDI) to India, despite a dip of 31.55%

e
to $11.77 billion?
(a) Mauritius (b) USA (c) Singapore (d) UAE
46.

n k
On May 19, 2024 India-born aviator and commercial pilot became the first space tourist from India. What is the

a
name of this individual who flew aboard a spacecraft of Blue Origin for a short recreational trip to space?

r
(a) Rakesh Patel (b) Anil Kumar

p
(c) Gopi Thotakura (d) Vivek Singh

o
1D
7F3A0B9C7Estatements
47. Which of thetr-5Efollowing about ISRO is incorrect?
(a) ISRO was established in 1969 to harness space technology for national development.

T
(b) ISRO's Mars Orbiter Mission (Mangalyaan) made India the first Asian nation to reach Martian orbit and the
first nation in the world to do so in its first attempt.
(c) ISRO developed the Geosynchronous Satellite Launch Vehicle (GSLV) series for launching heavier
payloads into geostationary orbit.
(d) ISRO's Chandrayaan-2 mission was the first mission to land on the Moon's south pole.
Passage (Q.48-Q.52): Household Consumption Expenditure Survey (HCES)
How do Indians divide their consumption expenditure across
7U1M categories? How many are below the poverty line?
tr-5Q7P3O0S9L
How has the picture changed over time? Answers to such questions have always been drawn from the
quinquennial consumption expenditure surveys. After a decade-long hiatus, a factsheet based on the revamped
and much awaited Household Consumption Expenditure Survey (HCES) for 2022-23 has been released.
The factsheet gives a useful glimpse into present patterns of household consumption. There have been quick
comparisons with earlier rounds of the consumption expenditure survey, especially with the last publicly
7U1Mround in 2011-12, to deduce a dramatic reduction in poverty over time and/or drastic changes in
tr-available 68th
5Q7P3O0S9L
consumption patterns. Such comparisons are premature — the full data has not been released and there are
several changes in item coverage, survey design and methodology.
The 2011-12 round covered 347 items of consumption compared to 405 in HCES. Such revisions have been
done in the past too and are par for the course as consumption habits change over time.
There are three major changes that make the current survey non-comparable with its predecessors. First, the
earlier rounds had one single questionnaire to cover expenditure on all items. The new HCES has three separate
Head Office: 127, Zone II, MP Nagar, Bhopal |+91-7676564400| https://www.toprankers.com Page 14 of 36
surveys on food items, consumable and service items and durable goods respectively. Second, the HCES
involved multiple separate visits to households.
As data collection methodologies, both these changes are welcome because it is well-known that respondent
fatigue with long questionnaires in a single sitting leads to poorer responses. Shorter and more focused
questionnaires elicit more precise answers. However, these changes have implications for comparability — these
are likely to yield higher estimates compared to previous rounds.
Extracted with edits and revisions from https://indianexpress.com
48. Who conducts the Household Consumption Expenditure Survey (HCES) and at what time interval is it
conducted?
(a) National Statistical Office (NSO), every 3 years
(b) National Sample Survey Office (NSSO), every 5 years
(c) National Statistical Office (NSO), every 5 years
(d) Ministry of Finance, every 10 years

c o m
.
49. Which of the following statements about the recent Household Consumption Expenditure Survey is incorrect?

rs
(a) The survey provides information on the typical spending on both goods (including food and non-food items)
and services.

higher average MPCE than the same category of population in the urban areas.

k e
(b) The bottom 5% of India’s rural population, ranked by Monthly Per Capita Consumer Expenditure, has a

n
(c) Sikkim has the highest Monthly Per Capita Consumer Expenditure in both rural and urban areas, while

a
Chhattisgarh has the lowest.

r
(d) In 2022-23, 46% of rural household expenditure and 39% of urban household expenditure were on food

p
items.

o
7E1D
50. tr-5E7F3A0B9Crecommendations led to the establishment of the National Statistical Commission (NSC)
Which commission's
in India?

51.
(a) Gadgil Commission
(c) Rangarajan Commission
T (b) Sarkaria Commission
(d) Malimath Commission
The book 'Poverty and Un-British Rule in India,' which marked the earliest attempts to estimate poverty based
on the cost of a subsistence diet, was written by a prominent Indian nationalist and leader. Who is this author?
(a) Bal Gangadhar Tilak (b) Gopal Krishna Gokhale
(c) Dadabhai Naoroji (d) Lala Lajpat Rai
52. Who is the current Vice Chairperson of NITI tr- 5Q7P3O0S
Aayog, 9L7U1M the rank and status of a Cabinet Minister, who took
holding
over the position on 1 May 2022?
(a) Rajiv Kumar (b) Arvind Panagariya
(c) Suman Bery (d) Bibek Debroy

7U1M
tr-5Q7P3O0S9L

Head Office: 127, Zone II, MP Nagar, Bhopal |+91-7676564400| https://www.toprankers.com Page 15 of 36
SECTION – C: LEGAL REASONING
Directions (Q.53-Q.84): Read the comprehension carefully and answer the questions.
Passage (Q.53-Q.57): False imprisonment is restraining a person in a bounded area without justification or
consent. It is a common law misdemeanor and a tort. For imprisonment, it is not necessary that the person should
be put behind bars, but he should be confined in such an area from where there are no possible ways of escape
except the will of the person who is confining the person within that area. It is not the degree of the imprisonment
that matters but it is the absence of lawful authority to justify unlawful confinement which is of relevance. The
defense to false imprisonment includes consent of the plaintiff or voluntary assumption of the risk, probable
cause and contributory negligence. Reasonable care and acting in good faith are no defenses for this tort. The
following are the major elements of False Imprisonment: (a) Period of Confinement - The tort of false
imprisonment arises whatever may be the period of confinement. An otherwise lawful detention may become

m
unlawful if the detention is prolonged for an unreasonable period of time. (b) The Intention Factor - Normally

o
the tort of false imprisonment must be intentional. A person is not liable for false imprisonment unless his or her

c
act is done for the purpose of imposing a confinement or with knowledge that such a confinement, to a substantial

rs .
certainty will result from it. Malice is irrelevant to this tort. Even negligent acts can qualify as false
imprisonment. (c) Knowledge of the Plaintiff - There is no requirement that the plaintiff alleging false
imprisonment was aware of the restraint on his freedom at the time of his confinement. If the person is confined

k e
in a room, with one of the entries known to the plaintiff closed, and the room has more than one entry-exit door,
but the plaintiff has no knowledge about the same, the defendant will still be held liable. Thus, the person

53. n
confined does not have to be aware of the confinement or be harmed by it as it is actionable per se.

a
(Source: https://www.lawctopus.com/academike/law-of-false-imprisonment-in-india/ )

r
Which of the following statement cannot be concluded from the given passage?
(a) In order to be considered
tr-5E7F3A0B9C

o
7E1D

p
imprisoned under criminal law, a person does not necessarily need to be placed
behind bars; rather, he need be kept in a space from which he is unable to flee.

T
(b) The relevant factor of false imprisonment is the absence of legal authority to support unlawful confinement,
not the severity of the imprisonment.
(c) It is not necessary for the plaintiff who is claiming false imprisonment to have known of the restriction on
his freedom when he was being held captive.
(d) The defendant will still be held accountable even if the person is imprisoned in a room with one of the entries
that the plaintiff knew about locked and the room has many entry-exit doors but the plaintiff is unaware of
any of them.
54. T planned to meet a friend at a café close totr-her home.
3O0S9L7U 1Mleft her daughter X, while she slept and locked the
She
5Q7P
front door from outside as directed by X. Within an hour, she was back at her house, where her daughter was
still sound asleep. X wanted to file a false imprisonment claim against her mother as she had been locked from
the outside when she woke up. Can she actually do that?
(a) She is ineligible to file a complaint because it is legitimate for a mother to confine the child in order to protect
her.
(b) Since her
7U1Mmother locked her up with the intention of preventing her from leaving the house, she is eligible
tr-5Q7P3O0S9L
to file a complaint.
(c) Due to the fact that she was asleep when she was restrained, she is ineligible to file a complaint.
(d) She is ineligible to file a complaint because she herself gave consent to be locked from the outside.

Head Office: 127, Zone II, MP Nagar, Bhopal |+91-7676564400| https://www.toprankers.com Page 16 of 36
55. R was teaching a class of 70 students about professional ethics. He answered a call in the interim and went
outside to speak. Additionally, he closed the gate from the outside before returning three hours later. The students
questioned him about his absence when he got back, and he replied that he had an urgent meeting with a friend.
One of the students later learned that R had closed the door for three hours for no reasonable cause, and he then
filed a false imprisonment complaint against his teacher, R. Comment.
(a) R shall not be held accountable because a teacher has the legal authority to isolate students as necessary.
(b) R will be held accountable for confining the students without a valid reason.
(c) R will be held accountable for wrongfully confining the students with a valid reason
(d) R shall not be held accountable because R's misconduct had no deleterious consequences for the students.
56. V was asked to look after his younger brother because both of his parents were away on vacation. Since he
couldn't go out, he threw a grand party at his house and invited all of his friends. He accidentally locked one of

m
the doors in which two of his friends namely X and Y were sleeping while he was showing his other friends the

o
rooms so they could get some rest after the party. He assumed both X and Y to be asleep when they didn't say

c
goodbye in the morning, but he didn't check and instead headed to the office after taking a bath. When V's parents

would be accepted?

rs .
entered the room and let the friends out, they reported V for false imprisonment. Analyze whether their allegation

(a) V did not lock them in the room with any malicious intent, so their claims would not be upheld.

k e
(b) Their claims would not be upheld because he intentionally locked them inside and was not aware of it.
(c) They will be granted their claims, and V will be held accountable as negligent acts can qualify as false
imprisonment.

an
(d) Since it is clear from the case's facts that V knew about his friend's confinement, their claims should not be
upheld.

r
57.
tr-5E
arrived, and 7F3A0B9C7E
they
p
L asked Z to come to her1Dhouse. Someone rang the doorbell while they were watching a movie. L's parents had

o
had asked her to study for her exams which were supposed to be held on the next day. She

T
asked her friend to hide in the living room for 10 to 20 minutes, to which Z agreed. L also locked the door from
outside. L's parents informed her that her grandmother had passed away and that they had to leave for Mumbai
right away. She forgot about her friend and left in haste. Z was taken out from the room after three days when
her parents attempted to locate her. L was sued by Z's parents for false imprisonment. Comment
(a) L will be held accountable because she restrained Z for an unreasonable period of time.
(b) L is not liable because she genuinely forgot about Z and did not want to confine her for such an extended
period of time.
(c) L is not liable because she already obtained Z's consent.
tr-5Q7P3O 0S9L7U1M
(d) L will be held accountable because she should have been more cautious in her actions.
Passage (Q.58-Q.62): The concept of “Office of Profit” refers to a position held by legislators, such as MPs or
MLAs that brings them some financial gain, advantage, or benefit. This can potentially put legislators in a
position where they may be susceptible to government influence and unable to fairly discharge their
constitutional mandate of holding the government accountable. The Office of Profit law is intended to enforce
the principle of separation of power between the legislature and the executive. The law does not clearly define
7U1M
tr-5Q7P3O0S9L
what constitutes an Office of Profit but the definition has evolved over the years with interpretations made in
various court judgments. An Office of Profit has been interpreted to be a position that brings to the office-holder
some financial gain, or advantage, or benefit. The amount of such profit is immaterial. In 1964, the Supreme
Court ruled that the test for determining whether a person holds an Office of Profit is the test of appointment.
Several factors are considered in this determination including factors such as: (i) whether the government is the
appointing authority, and (ii) whether the government has the power to terminate the appointment, and (iii)
whether the government determines the remuneration. Under the provisions of Article 102 (1) and Article 191
(1) of the Constitution, an MP or an MLA is barred from holding any Office of Profit under the central or state
Head Office: 127, Zone II, MP Nagar, Bhopal |+91-7676564400| https://www.toprankers.com Page 17 of 36
government, unless it has been made immune to disqualification by law. Holding the Office of Profit can lead to
the disqualification of said legislator (MP or MLA) on the recommendation EC. Parliament has also enacted the
Parliament (Prevention of Disqualification) Act, 1959, which has been amended several times to expand the list
of exemptions. In conclusion, the concept of 'Office of Profit' is a legal principle that aims to ensure that
legislators do not hold positions that may compromise their ability to hold the government accountable.
[Source: https://prsindia.org/theprsblog/explained-law-on-holding-an-‘office-of-profit’]
58. An MLA named A has been appointed as a secretary in the State Government by the State Government who
holds the office on the will of state and draw salary from state government. The Election Commission (EC) has
recommended that A be disqualified for holding an “Office of Profit.” EC has made such recommendation
considering all the factors referred in determining whether an office constitutes as an “Office of Profit”. A has
filed a petition against the EC's recommendation in the Delhi High Court. What is the likely outcome of this
situation?

of Profit.

c m
(a) The Delhi High Court will uphold the EC's recommendation and A will be disqualified as he holds an Office

o
(b) The Delhi High Court will find that the position of parliamentary secretary does not constitute an “Office of
Profit” and A will not be disqualified

rs .
(c) The Delhi High Court will rule that the “Office of Profit” law does not apply in this case as A's appointment

e
was made immune by state legislation
(d) The Delhi High Court will rule that the “Office of Profit” law is unconstitutional and A will not be

59.
disqualified

an k
A Member of Parliament (MP) named A has been appointed as an advisor to a Central Government-run

r
corporation. This positions brings more financial stability to A, as the salary for this position is ten times greater

p
than what he is receiving currently receiving as an MP. The MP's political opponents have filed a complaint with
the Election Commission 7E1D(EC) claiming that A's appointment violates the “Office of Profit” law, as the Central
tr-5E7F3A0B9C

o
Government has appointed and also holds the power to terminate A's appointment. They also allege that the

T
salary that the Central Government (CG) is paying to A for the position of advisor is also a determining factor.
What is the likely outcome of this situation?
(a) The EC will find that the appointment does not violate the “Office of Profit” law as the government's power
to terminate the appointment is not a determining factor.
(b) The EC will find that the appointment violates the “Office of Profit” law and A will be disqualified as the
government's power to appoint, pay salary and terminate the appointment is a determining factor.
(c) The EC will find that the appointment is in violation of the “Office of Profit” law, but A will not be
disqualified as the appointment was madetr-5Qimmune
7P3O0S9Lby law
7U1M
(d) The EC will find that the appointment does not violate the “Office of Profit” law and A will not be
disqualified
60. A Municipal Council Member named A has been appointed as the chairperson of a government-funded, non-
profit organization. The Election Commission (EC) has recommended that A be disqualified for holding an
“Office of Profit.” A has filed a petition against the EC's recommendation in the state High Court. What is the
tr-likely 0S9L7U1M of this situation?
5Q7P3Ooutcome
(a) The state High Court will uphold the EC's recommendation and A will be disqualified
(b) The state High Court will rule that the “Office of Profit” law does not apply in this case as A's appointment
was made immune by state legislation
(c) The state High Court will rule that the “Office of Profit” law is unconstitutional and A will not be disqualified
(d) The State High Court will find that the position of chairperson does not constitute an “Office of Profit” as
the position of Municipal Council Member do not come under the ambit of Office of Profit law.

Head Office: 127, Zone II, MP Nagar, Bhopal |+91-7676564400| https://www.toprankers.com Page 18 of 36
61. A state legislator named A has been appointed as a director on the board of a privately-owned corporation. The
company holds the power to appoint and remove directors on the board and they hold maximum shareholding
onto the corporation. The Election Commission (EC) has recommended that A be disqualified for holding an
“Office of Profit.” A has filed a petition against the EC's recommendation in the High Court. What is the likely
outcome of this situation?
(a) The High Court will uphold the EC's recommendation.
(b) The High Court will rule that the Office of Profit law does not apply in this case as A's appointment was
made immune by state legislation
(c) The High Court will find that the position of director on the board of a state-owned corporation does
constitute an “Office of Profit” as the private company holds authority in matter of this appointment
(d) None of the above

m
62. A head of the board of filming in the Ministry of Entertainment, named A, has been appointed to a position by
the government, the government pays him salary but do not hold power to terminate him. A has decided to

.c o
contest for a seat in parliament as an MP. However, the Election Commission (EC) has recommended that A
might get disqualified for holding an “Office of Profit”, if he wins the seat. Assuming that A wins the election

rs
and files a petition against the EC's recommendation in the Delhi High Court. What is the likely outcome of this
situation?

k e
(a) The Delhi High Court will uphold the EC's recommendation and A will be disqualified
(b) The Delhi High Court will find that the position of head of the board of filming does not constitute an “Office
of Profit” and A will not be disqualified

was made by the government.

r n
(c) The Delhi High Court will rule that the Office of Profit law does not apply in this case as A's appointment

a
(d) The Delhi High Court will rule that the Office of Profit law is unconstitutional and A will not be disqualified

o
7E1D

p
tr-5E7F3A0B9C Privity of contract is the rule that specifies only the parties directly involved in a contract
Passage (Q.63-Q.67):
can enforce the terms of the contract. It protects the parties from third-party interference. The rule is a common

T
law principle that essentially states that someone who isn’t a party to the contract can’t claim a right to the
benefits of the contract, nor can they enforce the obligations of the contract. Even if the third party might benefit
from the existence of the contract, they still can’t sue if they don’t receive such benefits. The Privity of the
contract came about when third parties went to court to enforce the terms of the contract, even though they
weren’t actually parties to the contract. Sometimes, however, it may appear that there’s no Privity of contract
between two parties, but there actually is.
Trust Relationships: Situations involving trust often 0Sserve as exceptions to Privity. In cases where a contract is
7U1M
tr-5Q7P3O 9L
made on behalf of or for the benefit of a third party, the third party may enforce claims under trust law. Even
though the trust beneficiary is not a party to the contract, he can sue the trustee if the trustee does not follow the
contract.
A promisee can be held to be a trustee for the third party only if there is an intention to create a trust by the
promisor, and this intention must be to benefit that third party. The intention to benefit the third party must be
tr-an irrevocable
5Q7P 3O0S9L7U1Mone, and a mere intention to confer a benefit is not enough; there has to be a construction (legal
existence) of trust. Trust is distinguishable from an intention to gift something to someone.

Head Office: 127, Zone II, MP Nagar, Bhopal |+91-7676564400| https://www.toprankers.com Page 19 of 36
63. X and Y entered into a contract for the sale of thirty bottles of Pepsi. X had to supply Y with the thirty bottles,
and Y were to sell them further to a group of tourists who wanted to drink something while on their way up to
the mountain top. Now, X was to take the bottles directly from a manufacturer, from a factory, who knew about
the contract between X and Y. X took the bottles, but on his way to Y, the bottles fell and broke there. He failed
to perform the contract and Y decided to sue the manufacturer. Can he do so?
(a) Yes, as the manufacturer was aware of the contract and so, he should pay the damages.
(b) No, because there was no contract between Y and the manufacturer, so, the latter cannot pay damages to the
former.
(c) According to the exception of trust here, he can be held liable for paying damages to Y.
(d) He should have manufactured the bottles nicely, and since he failed to do so, he will be held liable here.
64. In the previous question of X and Y, can the tourists sue X for failing to perform his contract?

m
(a) No, because they were not the parties to the contract and so they have no right on that front.
(b) Yes, because they were the ones who were being benefitted from the contract and so, they can.
(c) Yes, the exception of trust comes here, and they can sue X for his non-performance.
(d) It is on X and Y to decide who they can make the Privity and put the trust into.

.c o
rs
65. Riya was an orphaned teenager. She used to live with her uncle and aunt. The uncle and aunt were very nice
people and they provided her with everything that she needed, and she was living a luxurious life there. Her

k e
uncle contracted with her aunt that once he finds a suitable man for Riya, he will gift Riya a diamond necklace.
She was very happy, hearing this she was eager to get married. She got married but she did not get a diamond

thing?

r n
necklace and she was really angry. Can she sue her uncle and aunt because they failed to perform a contractual

a
(a) No, because she was not the party to the contract between her aunt and uncle.

(c) Yes, because


tr-5E7F3Ashe

o
0B9Cwas

p
(b) No, because there might have been a reasonable reason to not give her the necklace.
7E1Dthe beneficiary to the contract and so only, she can register a complaint.

(d) Yes, because she was the one with whom the uncle and aunt had built the trust so she could enforce the

66.
contract.

T
Gaurav was a big and renowned businessman. He always was overseas and had a lot of trips abroad. He had a
son named Karan. One time on his business trip, he fell in love with a woman named Andrea and had a son
named Jorge. Gaurav suffered from a heart attack and the chances of his living was meagre. So, he called his
son Karan and transferred all his assets to him, he also held him in trust that he must provide One thousand
dollars to Jorge every month without fail. After he passed away, Karan decided to not do that because he was
very angry at his father. Can Jorge bring on a suit to recover the money?
(a) No, because he was an illegitimate son, and 1M demand money from the legitimate son.
0S9L7Unot
tr-5Qhe could
7P3O
(b) No, because he was not a party to the contract and so, it does not matter even if he brings a suit.
(c) Yes, because there was a trust created and so he can bring a suit to recover.
(d) It was a dying wish and so, he can bring a suit because he was a party to the contract.
67. Amish and Devgan were two friends. Amish wanted to start a side business as a side hustle to earn more money.
He asked Devgan if he wanted to join him, so they could earn more money together, but Devgan said he was
7U1M
5Q7P3O0S9Lfrom
tr-suffering leukemia and couldn't do so. Amish decided to start the business alone. He had the intention of
sharing some of the profits with Devgan later. He discussed this idea with his business manager. However, he
did not follow through on his intention, and when Devgan found out, he sought to make a claim for the money
under trust created by Amish. Can he do so?
(a) No, because he was party to the contract; he was a mere beneficiary.
(b) No, because there was no trust created, there was only a mere intention to do so.
(c) No, since he was not any business partner, he would not get the profit.
(d) Yes, because there was an intention to do so, and Amish failed to do that and so Devgan can bring a suit.
Head Office: 127, Zone II, MP Nagar, Bhopal |+91-7676564400| https://www.toprankers.com Page 20 of 36
Passage (Q.68-Q.72): In Vishakhapatnam Gas Leak case, the NGT observed that apart from statutory liabilities,
the company is also liable to restore the damage caused to the environment. It thus reiterated the position that,
even in the absence of any statutory recognition, the ‘Polluter Pays Principle’ (PPP) is well enshrined in the
jurisprudence of Indian environmental law. The principle implies that he/she who damages the environment
should bear the cost of rectifying that damage. So according to the ‘Polluter Pays Principle’, the polluter has to
not only compensate the victims of pollution but also compensate for the restoration of environmental
degradation caused.
The observation made by the NGT is that the Polluter Pays Principle follows the principle of “absolute liability”
and not “strict liability”. The principle of strict liability laid down in the matter of Rylands v. Fletcher states that
certain activities are inherently risky and that those who engage in them should bear the responsibility for any
harm that result. It differs from absolute liability as established through MC Mehta v. Union of India. Whereas

m
strict liability allows exceptions if the liability has been accrued by an Act of God or act of third party, absolute
liability offers no exception to industries involved in hazardous activities, which are liable for the damage so
triggered notwithstanding adherence to the highest safety norms.

.c o
In the matter of MC Mehta and Anr. v. Union of India, the Supreme Court held that it was not bound to follow

rs
the 19th Century English rule of strict liability laid down in the case of Rylands v. Fletcher. The Court evolved
a rule which is suitable to prevail in the social and economic India of the present times.

Leak Tragedy: Why the NGT order is cause for concern” by Mantul Bajpai,]

k e
[Extracted, with edits and revisions from “Strict Liability v. Absolute Liability vis-a-vis the Visakhapatnam Gas

68.

an
Rivertown, which relies heavily on its river for drinking water and recreational activities. Rivertech Industries,

r
is a medium-sized manufacturing facility situated on the outskirts of Rivertown. Established several decades
ago, Rivertech Industries specializes in the production of hazardous chemical compounds used in various
industrial applications,7Eranging
tr-5E7F3A0B9C

o
1D

p
from cleaning agents to pesticides. However, recently, it was found to be illegally
dumping toxic waste into the river, contaminating the water and harming the ecosystem. The toxic waste

T
discharged by the factory directly into the river contaminates the water supply of Rivertown. Vineet, a citizen of
Rivertown who rely on the river water for drinking, cooking, and bathing were exposed to harmful chemicals,
leading to serious health problems such as gastrointestinal issues, skin disorders. Feeling aggrieved by the
situation, Vineet decided to take action against Rivertech Industries by filing a case in the National Green
Tribunal (NGT), decide whether Vineet is likely to succeed or not by applying the principle of PPP?
(a) Yes, because Rivertech Industries is illegally dumping toxic waste into the river, leading to contamination
of the water supply and harm to the ecosystem.
(b) No, because the legal process could be complex and lengthy, requiring substantial evidence and legal
7U1M
tr-5Q7P3O0S9L
expertise to prove Rivertech Industries' culpability beyond a reasonable doubt.
(c) Yes, because the 'Polluter Pays Principle' holds polluters accountable for the environmental damage they
cause, including compensating victims and restoring degraded ecosystems.
(d) No, because the NGT may require additional evidence to establish a clear causal link between Rivertech
Industries' activities and the harm suffered by Vineet.
69. In a city, Cleanville, a large industrial plant “Polluticon Industries” has been found responsible for leaking a
1M
5Q7P3O0S9L7U
tr-hazardous gas one day leading to health problems among nearby residents and environmental degradation. The
plant has been operating for years without adequate pollution control measures. Ashish, a citizen of Cleanville
and an environment advocate decides to take legal action against Polluticon Industries for the harm caused by
its pollution. However, Polluticon Industries claims that the gas was leaked because of the lightening which
ruptured the gas chamber. Decide who is likely to succeed by applying the principle of PPP?
(a) Ashish, because Polluter Pays Principle follows the principle of absolute liability instead of strict liability.
(b) Polluticon Industries, because Polluter Pays Principle follows the principle of strict liability allowing
exception if the liability has been accrued by an Act of God.
Head Office: 127, Zone II, MP Nagar, Bhopal |+91-7676564400| https://www.toprankers.com Page 21 of 36
(c) Ashish, because Polluticon Industries operated without adequate pollution control measures for years,
demonstrating a lack of diligence in preventing environmental harm.
(d) Polluticon Industries, because Polluticon Industries took reasonable precautions to prevent the gas leak.
70. In a city, Cleanville, a large industrial plant “Polluticon Industries”, is a sprawling complex located on the
outskirts of Cleanville. Established several decades ago, Polluticon Industries specializes in the manufacturing
of hazardous chemical products used in various industrial applications, including petrochemicals, plastics, and
solvents but with regular maintenance and inspections of its facilities. One day, a chemical spill from Polluticon
Industries resulted in significant harm to the environment and nearby residents. Will Polluticon Industries be
held liable under strict liability or absolute liability applying the principle of PPP?
(a) Yes, Polluticon Industries will be held liable under the strict liability because it is engaged in a work that is
inherently risky imposing responsibility on Polluticon Industries for the consequences of its activities.
(b) Yes, Polluticon Industries will be held liable under the absolute liability because Polluticon Industries is

proper maintenance and inspections.

c o m
involved in hazardous activities, which is liable for the chemical spill even if damage is caused after taking

.
(c) No, Polluticon Industries will not be held liable under the strict liability because the chemical spill resulted

rs
from an unforeseeable event beyond the company's control, such as a natural disaster or third party fault.
(d) No, Polluticon Industries will not be held liable under the strict liability because it can demonstrate that it

e
took all reasonable precautions to prevent the spill, including regular maintenance and inspections of its

k
facilities.

n
71. In the above instance, suppose Polluticon Industries claim that the chemical spill was exacerbated by a third-

a
party contractor hired by Polluticon Industries to transport hazardous materials. The contractor's negligence in

r
handling and transporting the chemicals contributed to the spill. In light of these circumstances, will Polluticon
Industries be held liable under the strict liability or absolute liability principle for the harm caused by the
chemical spill, considering
tr-5E7F3A0B9C

o
7E1D

p
the involvement of a third-party contractor?
(a) Polluticon Industries will be held liable under the strict liability because it is engaged in a work that is

T
inherently risky imposing responsibility on Polluticon Industries for the consequences of its activities.
(b) Polluticon Industries will be held liable under the strict liability because strict liability principle imposes
responsibility on Polluticon Industries for the consequences of its activities, regardless of intent or fault.
(c) Polluticon Industries will not be held liable under the strict liability but under the principle of absolute
liability as there exists no exception for the polluter.
(d) Polluticon Industries will not be held liable under the strict liability because it can demonstrate that it took
all reasonable precautions to prevent the spill, including regular maintenance and inspections of its facilities.
72. On one day, because of the carelessness of Production
tr-5Q7P3O0S9L
line
7U 1M staff of XYZ Chemicals, there is a spillage of the
hazardous gas because of which, numerous nuisances and bugs in the close by ranch are killed. There is no
physical injury caused to individuals living close by and the laborers of the Production line. In any case, the
spillage was humungous to such an extent that it diminished the nature of air in the city creating an unhealthy
environment for individuals living around nearby. For this situation,
(a) XYZ Chemicals will be absolutely liable but not strictly liable and also liable under Polluter Pays Principle.
(b) XYZ Chemicals will not be liable under the Strict and Absolute liability principles because the leakage only
3O0S9L7U1M
tr-5Q7Pkilled the pests and insects.
(c) XYZ Chemicals liability under the Strict and Absolute liability principles will depend upon the inquiry as to
whether the leaked gas was a hazardous substance/activity or chemical or not and; XYZ will not be liable
under the Polluter Pays Principle.
(d) XYZ Chemicals will not be liable under the Polluter Pays Principle as the incident happened because of the
carelessness of Production line staff of XYZ Chemicals.

Head Office: 127, Zone II, MP Nagar, Bhopal |+91-7676564400| https://www.toprankers.com Page 22 of 36
Passage (Q.73-Q.77): The Supreme Court quashed the broadcast ban placed on MediaOne TV and cautioned
the Union against sealed cover submissions.
The Judgement held that the Union could not use unsubstantiated ‘national security concerns’ to violate press
freedom and the freedom of speech. Art 19(1) (a) which is Right to Freedom of Speech and Expression is not an
absolute right and can be restricted on grounds of National Security, provided under Art 19(2) among others.
MediaOne TV is reportedly operated by the Jamaat-e-Islami Hind (JEIH), a socio-religious organisation that is
involved in a number of academic, faith-based and policy projects. In January 2022, the Ministry of Information
and Broadcasting (I&B Ministry) refused to renew MediaOne TV’s transmission license, barring them from
continuing operations. The I&B Ministry cited national security concerns, indicating that the Union had issues
with the content aired by the channel.
The broadcast ban was immediately challenged at the Kerala High Court. However, the Union’s submissions

m
were made through a ‘sealed cover’—only the Bench was privy to the material that formed the basis of the ban.

o
MediaOne was denied access. In February 2022, the Kerala HC upheld the ban and stated that the Union’s sealed
cover submissions showed that the national security concerns were justified.

c
rs .
In March 2022, MediaOne TV challenged the Kerala HC’s decision at the Supreme Court. The Bench held that
the broadcast ban was unjustified as there was no evidence supporting the Union’s claims that MediaOne posed

e
a threat to national security. However, the Bench stated that JEIH is not a banned organisation and that there was

k
nothing to suggest that JEIH posed a national security risk.
Having reviewed the sealed cover material, the SC noted that the HC provided ‘no explanation’ to justify its

r n
decision. The Bench criticised the sealed cover submissions as well for compromising fair proceedings by

a
denying MediaOne the opportunity to defend themselves.
It admitted that there are situations where national security concerns will override the rights of petitioners and
the principles of natural
tr-5E7F3A0B9C

o
7E1D

p
justice. However, this is not one such case.
[Extracted, with edits and revisions from “Judgement Pronouncement: MediaOne Broadcast Ban” by Ajoy

T
Karpuram, Supreme Court Observer]
73. Kal Tak is an Indian leading Hindi-language news channel owned by TV Today Network, part of the New Delhi–
based media conglomerate Living Media group. In order to renew its transmission license, Kal Tak filed an
application with the Ministry of Information and Broadcasting. I&B Ministry declined the application based on
enmity it had with the head of the media group and banned from continuing its airing claiming that the content
air by the channel is instigating people to revolt against the government of New Delhi. Decide
(a) The ban by I&B Ministry is valid as Freedom of Speech & Expression can be restricted on grounds of
National Security 7U1M
tr-5Q7P3O0S9L
(b) The ban by I&B Ministry is invalid because Freedom of Speech & Expression can’t be restricted on grounds
of National Security
(c) The ban by I&B Ministry is invalid as Freedom of Speech & Expression can be restricted on grounds of
National Security, which is missing in this case.
(d) The ban by I&B Ministry is valid as Union had issues with the content aired by the channel.
5Q7P3O0S9L 7U1M
74. tr-News18 India isan Indian Media television channel owned by Network 18. It was launched in 1970 as Channel
7 by Jagran Prakashan Limited and is watched all over the country by a majority population. On 26/11/2008, the
day marked a starting of series of terrorist attacks that took place when 10 members of Lashkar-e-Taiba, a militant
Islamist organisation from Pakistan, carried out 12 coordinated shooting and bombing attacks lasting four days
across Mumbai. In the immediate aftermath of the attacks, the Ministry of Information and Broadcasting issued
banning notice to News18 for telecasting live the attacks as well as strategies of the Police on the National
Television in crisis situations. The ban was challenged on the ground for violating Right to Freedom of Speech
and Expression. Decide
Head Office: 127, Zone II, MP Nagar, Bhopal |+91-7676564400| https://www.toprankers.com Page 23 of 36
(a) The ban is valid as it is in interest of National Security
(b) The ban is valid as it was followed by a notice following the due process
(c) The ban is invalid because it infringed the Art 19(1)(a)
(d) The ban is invalid because News18 have a Right to telecasting live the attacks
75. News18 India is an Indian Media television channel owned by Network 18. It was launched in 1970 as Channel
7 by Jagran Prakashan Limited and is watched all over the country by a majority population. On 26/11/2008, the
day marked a starting of series of terrorist attacks that took place when 10 members of Lashkar-e-Taiba, a militant
Islamist organisation from Pakistan, carried out 12 coordinated shooting and bombing attacks lasting four days
across Mumbai. In the immediate aftermath of the attacks, the Ministry of Information and Broadcasting issued
banning notice to News18 on grounds of National Security. News18 challenged the ban at Bombay High Court
to which the Union submitted ‘sealed cover’ reasons, not disclosing such information to News18. Choose what
will be the judgment of Bombay HC on the basis of Kerala High Court’s ruling following such submission by
the Union.

o m
(a) The ban is valid as the Right to Freedom of Speech and Expression is an absolute right and cannot be

c
.
restricted

rs
(b) The ban is valid as the Union’s sealed cover submissions showed that the national security concerns were
justified

freedom and the freedom of speech

k e
(c) The ban is invalid as Union could not use unsubstantiated ‘national security concerns’ to violate press

n
(d) The ban is invalid as the sealed cover reasons compromise fair proceedings by denying News18 the
opportunity to defend themselves
76.

r a
Kal Tak is an Indian leading Hindi-language news channel owned by TV Today Network, part of the New Delhi–

p
based media conglomerate Living Media group. In order to renew its transmission license, Kal Tak filed an

o
applicationtr-with
5E7F3Athe 7E1D
0B9CMinistry of Information and Broadcasting. I&B Ministry declined the application and
banned from continuing its airing claiming that the content aired by the channel is against national security.

T
KalTak challenged the ban at Bombay High Court to which the Union submitted ‘sealed cover’ reasons, not
disclosing such information to News18. Decide the judgment on the basis of ruling of the Apex Court.
(a) The ban by I&B Ministry is valid as Freedom of Speech & Expression can be restricted on grounds of
National Security
(b) The ban by I&B Ministry is invalid because Freedom of Speech & Expression can’t be restricted on grounds
of National Security
(c) The ban by I&B Ministry is invalid as Freedom of Speech & Expression is an absolute right
(d) The ban by I&B Ministry is invalid astr-5Q sealed cover
7P3O0S9L 7U1M submissions cannot be used to provide ‘blanket

immunity’ to the Union


77. Consider the following statement, decide which of the following is true according to the passage?
(a) Art 19 is a restricted right subject to Art 19(2) only
(b) Sealed cover submissions is a fair process to protect Security of the State
(c) National security concerns cannot override the principles of natural justice
3O0S9L7U1M
5Q7PNot
tr-(d) providing an opportunity to be heard can violate fair proceedings

Head Office: 127, Zone II, MP Nagar, Bhopal |+91-7676564400| https://www.toprankers.com Page 24 of 36
Passage (Q.78-Q.84): In a writ petition where the primary issue was the ownership of the land, the Calcutta
High Court held that this dispute should be resolved by a civil court, not a writ court. The Court affirmed that
while illegal, irrational, or malafide actions of the State or its instrumentalities are open to judicial review, a writ
court should not be used to resolve private disputes, contractual issues, or questions of ownership.
The Court stated that the disputed questions of fact cannot be decided in a writ petition. The Court said that the
Judicial review is applicable to actions that are illegal, contravene prescribed procedures, are unreasonable,
irrational, or mala fide. However, private disputes, contractual disagreements, or questions of fact regarding land
ownership should be resolved by the Civil Court, not the Writ Court.
“…every action of State or its instrumentality, which is illegal, in contravention of the prescribed procedure,
unreasonable, irrational or malafide is open to judicial review. Every executive or administrative action of the
State or other statutory or public bodies, “legally treated to be authority”, which is violative of fundamental rights
or any statute, is open to judicial review.”

o m
Given the ongoing dispute and the need to determine land ownership, the Court dismissed the writ petition and

c
.
held that the petitioner is not entitled to any relief.

rs
[Extracted with revisions from https://www.scconline.com/blog/post/2023/10/19/disputed-question-of-fact-
cannot-be-decided-in-writ-petition-calcutta-high-court-scc-blog-legal-research/ ]
78.

e
In a case involving a dispute over land title of property, which court would be appropriate for resolving the issue,
according to the principle provided?

k
79.
(a) Criminal Court (b) Writ Court

r an
(c) Civil Court (d) Arbitration Court
Which type of actions of the State or its instrumentalities are open to judicial review, as per the principle

p
provided?
(a) Actions related to 7E
private contracts (b) Actions that are illegal, irrational, or malafide

o
1D
tr-5E7F3A0B9C
(c) Actions involving personal disputes (d) Actions related to property taxation
80.

T
X is the owner of a local business which makes fruit liquor from orchards in Himachal Pradesh. For this purpose,
X had obtained a liquor license from the government. Emily, a government official, arbitrarily revoked the
license of X without following the prescribed procedures. The business owner filed a writ petition challenging
the revocation. Based on the principle provided, is Emily's action subject to judicial review?
(a) Yes, because Emily's action was illegal and contravened prescribed procedures.
(b) No, because government officials have discretion in revoking licenses.
(c) Yes, because private disputes between individuals are open to judicial review.
1M
(d) No, because the revocation of a license constitutes a7U
tr-5Q7P3O0S9L private dispute between X and Emily.
81. John and Sarah entered into a private contract for the sale of land. Before the sale, Sarah had to ensure that the
boundary wall of the property is fixed to prevent illegal encroachment or trespass on the land. Once the boundary
wall was built, John would transfer the money and start building a public school on the land. However, Sarah
failed to fulfil her obligations under the contract, leading to a dispute between them. John filed a writ petition
seeking resolution
7U1M
of the contractual disagreement. Based on the principle provided, is John's contractual dispute
tr-5Q7P3O0S9L
with Sarah subject to judicial review?
(a) Yes, because contractual disagreements are open to judicial review.
(b) No, because private disputes and contractual disagreements are not subject to judicial review.
(c) Yes, because the dispute involves fundamental rights violation as it concerns the construction of a public
school
(d) No, because the question of the dispute is about ownership of land

Head Office: 127, Zone II, MP Nagar, Bhopal |+91-7676564400| https://www.toprankers.com Page 25 of 36
82. The city council passed a resolution to demolish a historical building without providing any justification or
following the prescribed procedures. A group of concerned citizens filed a writ petition challenging the council's
decision. Based on the principle provided, is the city council's resolution subject to judicial review?
(a) Yes, because actions that contravene prescribed procedures are open to judicial review.
(b) No, because the city council has discretion in making decisions about buildings.
(c) Yes, because private disputes between individuals are subject to judicial review.
(d) No, because the concerned citizens did not exhaust administrative remedies before filing the writ petition.
83. David and Emma, neighbours in a residential area, have been engaged in a longstanding dispute over the height
of the fence separating their properties. David filed a writ petition seeking resolution of the dispute, claiming
that Emma's fence violates his property rights. Based on the principle provided, is David's dispute with Emma
over the fence subject to judicial review?

m
(a) Yes, because disputes between neighbours are open to judicial review.
(b) No, because private disputes, including neighbourly disagreements, are not subject to judicial review.
(c) Yes, because the dispute involves a violation of fundamental rights.
(d) No, because land ownership has to be decided by a writ court

.c o
rs
84. Sarah, a government official, issued a notice of violation to a local business for allegedly operating without a
proper license. The business owner disputed the allegation, claiming that they had obtained all necessary permits

k e
and licenses. In response, Sarah arbitrarily imposed fines on the business and ordered its closure. The business
owner filed a writ petition challenging Sarah's decision. Based on the principle provided, is the business owner's

n
dispute with Sarah over the alleged violation subject to judicial review?

a
(a) Yes, because every dispute between government officials and businesses are open to judicial review.

r
(b) No, because disputes regarding the validity of permits and licenses are not subject to judicial review.

(d) No, because 0B9C7E


there
tr-5E7F3A

o
is 1D
p
(c) Yes, because the dispute involves an arbitrary actions taken on part of Sarah.
no malafide or illegality apparent on the part of the state

T
7U1M
tr-5Q7P3O0S9L

7U1M
tr-5Q7P3O0S9L

Head Office: 127, Zone II, MP Nagar, Bhopal |+91-7676564400| https://www.toprankers.com Page 26 of 36
SECTION D: LOGICAL REASONING
Directions (Q.85-Q.108): Read the passage carefully and answer the questions.
Passage (Q.85-Q.90): In most developed countries, men have higher salaries, on average, than women. Much
of the salary differential results from the tendency of women to be in lower-paying occupations. The question of
whether this occupational employment pattern can be attributed to sex discrimination is a complex one. In fact,
wage differentials among occupations are the norm rather than the exception. Successful athletes commonly earn
more than Nobel Prize-winning academics; gifted artists often cannot earn enough to survive, while mediocre
investment bankers prosper. Given such differences, the question naturally arises: talent and ability being equal,
why does anyone—man or woman—enter a low-paying occupation? One obvious answer is personal choice. An
individual may prefer, for example, to teach math at a modest salary rather than to become a more highly paid
electrical engineer.

m
Some people argue that personal choice also explains sex-related wage differentials. According to this

o
explanation, many women, because they place a high priority on parenting and performing household services,

c
choose certain careers in which they are free to enter and leave the work force with minimum penalty. They may

rs .
choose to acquire skills, such as typing and sales clerking, that do not depreciate rapidly with temporary absences
from the work force. They may avoid occupational specialties that require extensive training periods, long and

e
unpredictable hours, and willingness to relocate, all of which make specialization in domestic activities
problematic.

n k
By choosing to invest less in developing their career potential and to expend less effort outside the home, women

a
must, according to this explanation, pay a price in the form of lower salaries. But women cannot be considered

r
the victims of discrimination because they prefer the lower-paying occupations to higher paying ones. An

p
alternative explanation for sex-related wage differentials is that women do not voluntarily choose lower-paying
occupations but are forced
7E1D into them by employers and social prejudices. According to proponents of this view,
tr-5E7F3A0B9C

o
employers who discriminate may refuse to hire qualified women for relatively high-paying occupations. More

T
generally, subtle society-wide prejudices may induce women to avoid certain occupations in favor of others that
are considered more suitable. Whether the discrimination is by employers in a particular occupation or by society
as a whole is irrelevant; the effect will be the same. Further, if such discrimination does occur, women excluded
from certain occupations will flood others, and this increase in supply will have a depressing effect on wages in
occupations dominated by women.
85. The author is likely to agree with each of the following EXCEPT that:
(a) Women may choose lower-paying occupations to have flexibility for parenting and household
7U1M
responsibilities. tr-5Q7P3O0S9L
(b) Wage differentials among occupations are common and not limited to gender differences.
(c) Women who prioritize career development and invest significant effort outside the home are likely to face
discrimination.
(d) The concentration of women in certain occupations may depress wages in those fields due to an oversupply
of workers.
1M
5Q7P3O0S9L7U
tr-Which
86. of the following correctly represents the passage?
(a) The fact that women sometimes opt for lower-paying occupations to accommodate parenting and household
responsibilities and that they, during other times, are steered into lower-paying jobs by conventional
prejudices explore the complex interplay of factors in contributing to the gender wage gap.
(b) Women are steered into lower-paying jobs by the complex interplay of employers' biases and societal
prejudices that restrict their access to higher-paying occupations, resulting into the gender wage gap and our
belief that women are not capable of high-paying jobs.

Head Office: 127, Zone II, MP Nagar, Bhopal |+91-7676564400| https://www.toprankers.com Page 27 of 36
(c) With more women joining the workforce, the increased availability of workers has made the interplay of
factors contributing to gender wage gap complex and difficult to understand, and it requires careful study.
(d) The interplay of factors in contributing to the gender wage gap has become more complex with more women
joining the workforce, thereby making employers' biases and societal prejudices a more important factor in
determining gender wage gap in developed countries.
87. Which of the following, if true, will strengthen the argument by those who advocate that women choose lower-
paying occupations to accommodate parenting and household responsibilities?
(a) According to Some people, it is the inherent nature of women to prioritize household responsibilities over
career responsibilities.
(b) The choice of women to accommodate parenting and household duties don’t themselves result from social
prejudices.

m
(c) Most women choose an option which is more convenient to them in the long run, taking care of their health
and career both.

responsibilities.

.c o
(d) Social prejudices against women result from women’s choice to accommodate parenting and household

rs
88. If the information in the passage is true, which of the following is likely to be true?
(a) The gender wage gap in developed countries would disappear if societal prejudices were eliminated.

expectations.

k e
(b) Women who choose to specialize in domestic activities do so without any influence from societal

qualifications.

r n
(c) Societal prejudices may lead women to avoid high-paying occupations even if they have the necessary

a
(d) Unlike what people think, societal prejudices do not play a significant role in the career choices of women

89.
in developed countries.
tr-5E7F3A0B9C

o
7E 1D

p
The author is likely to agree with which of the following?

T
(a) Women’s preference for careers with flexible hours and minimal training requirements explains why they
earn less than men on average.
(b) Occupational wage differentials are common, and personal preferences are the main reason women earn less
than men.
(c) Employers and societal biases are the primary reasons women are forced into lower-paying jobs, resulting
in a significant gender wage gap.
(d) The gender wage gap in developed countries is influenced by both personal choices and systemic
discrimination, making it a complex issue to attribute solely to one factor.
0S9L7U1Mtr-5Q7P3O
90. The argument that both personal choices and societal prejudices contribute to the gender wage gap is most closely
paralleled by which of the following arguments?
(a) The fact that consumers sometimes buy local products to support their community and that they, during other
times, are steered into buying local products due to lack of access to larger markets.
(b) The fact that students sometimes choose to attend less prestigious universities to stay close to home tells the
tr-5Q7Pstory behind
3O0S9L 7U1M their educational outcomes.
(c) Both the preference for personal time and lack of full-time job opportunities explore the complex interplay
of factors in influencing purchasing decisions.
(d) The fact that tourists sometimes visit nearby attractions to save money greatly influences their travel plan.

Head Office: 127, Zone II, MP Nagar, Bhopal |+91-7676564400| https://www.toprankers.com Page 28 of 36
Passage (Q.91-Q.96): Three decades after India's last penicillin manufacturing unit was shut down, the country
will start producing this active pharmaceutical ingredient (API or bulk drug) used in several antibiotics.
According to the Union Ministry of Health and Family Welfare (MoHFW), production of penicillin-G (or pen-
G) will resume this year.
This was phased out in the 1990s, when the country’s markets were flooded with cheaper alternatives, largely
from China. The decline in API production was noticed only in a few circles until late 2019, when supply chains
were disrupted following China’s stringent regulations on its industry. The Covid pandemic made the problem
grave and API shortages threatened to have serious ramifications outside India’s borders, given the country’s
status as the largest manufacturer of generic medicines. The resumption of penicillin manufacturing owes in
great measure to the government’s production-linked incentive (PLI) scheme as Pen-G manufacture is
cost-intensive and involves a complex fermentation and extraction process. That’s why drug
manufacturers find it prudent to outsource their production. The situation has compounded in the last few

o m
years because Chinese penicillin makers have been producing well below their capacity. In 2019, the public
sector Hindustan Antibiotics Ltd was reportedly the government’s first choice to restart its production under the

c
.
Make in India scheme. However, the PSU expressed its inability to participate in the venture, citing resource

rs
constraints. About the same time, the Department of Health Research informed the MoHFW that India needs
more than 13,000 million doses of penicillin in the next three years to deal with bacterial infections that cause

e
rheumatic fever – India has amongst the highest death rates from such illnesses. The government also received

k
requests from doctors to procure this bulk drug. Broad-spectrum antibiotics, such as azithromycin, that have
been used as penicillin substitutes are known to harm essential bacteria naturally present in the human body,
leaving a patient vulnerable to harmful germs.

r an
The PLI scheme envisages a support of 20 per cent for the first four years, 15 per cent for the fifth year, and 5

p
per cent for the sixth year on eligible sales of fermentation-based bulk drugs and hormones such as insulin. It’s

o
early yearstr-for the scheme
5E7F3A0B9C7E
1D and India still imports close to 90 per cent of all APIs for antibiotics. The challenge
for the country’s health authorities will be to ensure that the focus on self-reliance does not affect the affordability

91. T
of medicines. They should also make sure that the companies can sustain themselves once the government hand
holding is over.
Which of the following is an assumption behind the following statement:
“The resumption of penicillin manufacturing owes in great measure to the government’s production-linked
incentive (PLI) scheme as Pen-G manufacture is cost-intensive and involves a complex fermentation and
extraction process. That’s why drug manufacturers find it prudent to outsource their production.”
(a) Penicillin is an important Active pharmaceutical Ingredient (API) that benefits that benefits the citizens of
7U1M
India. tr-5Q7P3O0S9L

(b) Costs of producing Penicillin in India with the benefits of PLI should be less than that of outsourcing
Penicillin production.
(c) Raw material for producing Penicillin in India is cheaper than that available in the other countries.
(d) Costs of producing Penicillin in India with the benefits of PLI is more than that of outsourcing Penicillin
production.
7U1M
tr-5Q7P3O0S9L
92. Which of the following, if true, would most strengthen the argument that the PLI scheme will significantly
contribute to India's self-reliance in API production for antibiotics?
(a) The PLI scheme offers financial incentives to companies investing in advanced API manufacturing
technology.
(b) Few multinational pharmaceutical companies have expressed interest in the PLI scheme for API production.
(c) The cost of API production in India is projected to decrease by 25% due to the PLI scheme.
(d) There has been a significant increase in the number of skilled workers available for API production.

Head Office: 127, Zone II, MP Nagar, Bhopal |+91-7676564400| https://www.toprankers.com Page 29 of 36
93. Which of the following, if true, would most seriously weaken the claim that resuming production of penicillin
in India will address the country's vulnerability to API shortages?
(a) The global demand for penicillin is expected to decline over the next decade.
(b) India lacks the infrastructure to produce penicillin at a scale competitive with imports.
(c) The majority of penicillin production technology patents are held by foreign companies.
(d) Companies manufacturing in India will have to import majority of resources from other countries.
94. What can most reasonably be inferred about the impact of China's stringent regulations on its penicillin
production on the global API market?
(a) It led to a significant increase in the global prices of penicillin.
(b) It caused a worldwide shortage of penicillin, affecting many countries' healthcare systems.
(c) It prompted other countries to reconsider their reliance on Chinese penicillin.

m
(d) It accelerated the research and development of synthetic alternatives to penicillin.
95.
penicillin production?

.c o
Which of the following best represents the conclusion of the argument regarding the need for India to resume

(a) Resuming penicillin production is crucial for India to reduce its API import dependency.

rs
(b) India needs to restart penicillin production to address the high death rates from rheumatic fever.
(c) The resumption of penicillin production in India is essential for global health security.

e
(d) India's penicillin production is necessary to ensure the affordability of antibiotics domestically.

k
n
96. What is the main point of the passage regarding the relationship between the closure of India's penicillin
manufacturing units and the subsequent reliance on imports?

r a
(a) The closure led directly to an increase in healthcare costs due to reliance on imports.
(b) The reliance on imports exposed India to vulnerabilities in global supply chain disruptions.
(c) Importing APIs has

p
significantly improved the quality of antibiotics available in India.

o
7E1D
tr-5E7F3A0B9C
(d) The shutdown made India completely dependent on foreign countries for its penicillin supply.

T
Passage (Q.97-Q.102): Oilseeds and pulses farmers today suffer policy discrimination on two counts. The first
is from their minimum support prices (MSP) being largely on paper — unlike with rice, wheat and sugarcane,
where the government assures MSP either through large-scale direct procurement, or by forcing mills to pay.
Mustard is currently selling at Rs 5,000-5,100 per quintal in Rajasthan and chana (chickpea) at Rs 4,700-4,800
in Maharashtra, as against their respective MSPs of Rs 5,650 and Rs 5,440. And this is even before the crops
now in the field are to be marketed in about two months’ time. The second bias has to do with imports. Wheat,
milled rice, and sugar attract import duties. ,
7U1M
tr-5Q7P3O0S9L
It is not for nothing that India’s edible oil imports have risen from 11.6 million to 16.5 million tonnes (mt)
between 2013-14 and 2022-23, with the latter valued at $16.7 billion and meeting almost two-thirds of the
domestic consumption requirement. That’s not a record the Narendra Modi government would be terribly pleased
about. Things have been better in pulses, where imports have fallen from a peak of 6.6 mt in 2016-17 to 2.5 mt
in 2022-23, translating into a 90 per cent-plus self-sufficiency ratio. But the current year has seen a resurgence
of imports, with the Modi government swinging from a pro-producer to pro-consumer stance ahead of national
7U1M
5Q7P3O0S9LPrice
tr-elections. and tariff support apart, oilseeds and pulses have not received research and development attention
anywhere near that for wheat, rice or sugarcane. Denial of approval for genetically-modified hybrid mustard and
herbicide-resistant soyabean technologies is a manifestation of this official indifference. Even in pulses, the
breakthroughs have been largely limited to breeding of short-duration and photo-thermo insensitive varieties in
chana and moong.
It’s not surprising then — and welcome, in fact — that various departments within the government are flagging
concerns over the extant MSP and procurement policy favouring only a few crops, as a report in this newspaper
Head Office: 127, Zone II, MP Nagar, Bhopal |+91-7676564400| https://www.toprankers.com Page 30 of 36
has revealed. Farmers, like all rational economic actors, respond to price signals and incentives. MSPs should
supplement, not supplant markets. The two sub-sectors of Indian agriculture that have registered the highest
growth over the last two decades — livestock and horticulture — are the ones that are the most market-led. The
crops sub-sector, on the other hand, has exhibited less dynamism, in which MSP-based market distortions have
certainly played a role. That needs correction. The best way to do it is by replacing all price-based supports,
whether MSP or input subsidies, with per-acre income transfers.
97. What can be inferred about the impact of the government's current MSP and procurement policies on the diversity
of crops cultivated by farmers?
(a) Farmers are encouraged to grow a wider variety of crops due to MSP guarantees.
(b) The focus on rice, wheat, and sugarcane reduces incentives to cultivate other crops.
(c) MSP policies have led to an increased production of oilseeds and pulses nationwide.

m
(d) Government procurement policies have little to no impact on farming decisions.
98.
oilseeds and pulses is detrimental to India's agricultural self-sufficiency?

.c o
Which piece of evidence, if true, would most strengthen the argument that the policy discrimination against

(a) A recent study shows that farmers have reduced production of crops not favoured by the government.

rs
(b) Data indicating that the per acre yield of oilseeds and pulses has steadily declined.
(c) Evidence that oilseeds and pulses require less water compared to wheat and rice.

e
(d) Surveys revealing that farmers are satisfied with the current MSP for wheat and rice.

k
n
99. What information could most effectively weaken the argument that the high import duties on wheat, rice, and

a
sugar successfully protect domestic farmers of these crops?

r
(a) Domestic production of wheat, rice, and sugar has seen a consistent increase.

p
(b) Import duties have led to higher prices for consumers without benefiting farmers.
(c) International markets7E1Doffer lower prices for wheat, rice, and sugar than domestic markets.
tr-5E7F3A0B9C

To
(d) Farmers have reported a decrease in their net income despite no high import duties.
100. Based on the passage, what conclusion can be drawn about the effectiveness of price-based supports versus per-
acre income transfers in achieving a more dynamic crops sub-sector?
(a) Price-based supports are essential for maintaining the agricultural sector's stability.
(b) Per-acre income transfers would incentivize farmers to diversify their crop production.
(c) Price-based supports ensure a fair market price for all types of crops.
(d) Per-acre income transfers have little impact on the cultivation choices of farmers.
101. Given the bold statement that "MSPs should supplement, not
7U 1M supplant markets," what is the underlying principle
tr-5Q7P3O0S9L
regarding the role of government in agriculture as suggested by the passage?
(a) The government should primarily focus on regulating agricultural markets to ensure fairness.
(b) Government intervention in agriculture should ensure market stability and farmer welfare.
(c) The government's role is to provide safety nets, not to control agricultural market dynamics.
(d) Government policies should actively shape agricultural markets to promote crop diversity.
102. tr-The
5Q7Pauthor is1M
3O0S9L7U likely to agree with each of the following EXCEPT that
(a) Import of edible oils from other countries do not reflect well on the performance of government.
(b) Unlike wheat, milled rice, and sugar, Oilseeds and pulses do not attract import duties.
(c) Farmers prefer to produce crops whose production is incentivised by the government.
(d) Wheat, milled rice, and sugar are among the sub-sectors of Indian agriculture that are market-led.

Head Office: 127, Zone II, MP Nagar, Bhopal |+91-7676564400| https://www.toprankers.com Page 31 of 36
Passage (Q.103-Q.108): The death of a second Indian in the Russia-Ukraine conflict, which the Ministry of
External Affairs (MEA) confirmed, is proof of the toll the situation is taking on Indians who have been lured
into working with the Russian military. The story of dozens of Indians stuck at the frontlines of the war — it was
broken in a series of reports by The Hindu — prompted government agencies to crack down on recruiters in
India this week, with the Central Bureau of Investigation (CBI) raiding visa recruiters across India and filing
cases against them for human trafficking and for duping people into travelling to Russia where they were coerced
into dangerous jobs. Many of them were duped by online video advertisements that promised “safe jobs” with
the military far from the frontlines, even indicating more lucrative jobs in the neighbouring European Schengen
States. The MEA has now issued advisories and statements with appeals not to be “swayed” by such offers. After
videos emerged of Indians stationed in areas seeing live action along the front between Russia and Ukraine in
recently occupied territories, the government and India’s Embassy in Moscow also said they are working with
the Russian Army for early discharges to bring the Indians home.

o m
The government’s actions are much delayed and cold comfort to the families of the two men, from Surat and
from Hyderabad, whose bodies are yet to be repatriated. Others have been seriously injured. What is most

c
.
surprising is that the government has only spoken about the dangers of the Russian offers for military jobs now,

rs
when it is clear that the online ads and recruitments have been taking place for months. The government seems
to have been made aware of the problem only after a Member of Parliament from Hyderabad wrote to the MEA

k e
in January, asking for assistance in bringing some of his constituents back. Since then, officials have given vague
accounts of the numbers of those (“about 20”) who may have already travelled to danger zones. Now, after the

n
CBI raids, they have identified at least 35 men. Accounts from those in Russia put the number at well over 100.
The government needs to explain why it has not been able to make the case to bring back all the men more

r a
forcefully with the Russian authorities, despite good relations with Moscow. It has also dodged questions on
why it has simultaneously green-lighted a massive recruitment drive to Israel, another conflict zone. A more
structured, transparent

p
and humane approach will be required to make it clear, domestically, and to partners

o
7E1D
tr-5E7F3A0B9C
abroad, that Indian lives matter. The priority is to bring them home safely, but also to ensure that others are not

T
put in harm’s way, driven by the difficult economic situation back home.
103. Which of the following is the primary argument of the author?
(a) The Indian government faces criticism for its delayed response to the plight of Indian nationals trapped in
dangerous military jobs in the Russia-Ukraine conflict.
(b) The Indian government needs to improve its response to protect its nationals from being lured into dangerous
jobs in conflict zones and ensure their safe return.
(c) Indian nationals working in conflict zones are primarily motivated by the promise of lucrative employment
7U1M
in European countries. tr-5Q7P3O0S9L
(d) The Indian government has effectively prevented its nationals from joining foreign militaries by cracking
down on recruitment agencies.
104. Given the delayed government action on the recruitment of Indians by the Russian military, which criteria would
best assess the effectiveness of the government's response to this issue?
(a) The number of advisories issued by the government to the public against such recruitments.
3O0S9L7U1M
5Q7PThe
tr-(b) reduction in online advertisements promising lucrative jobs in conflict zones.
(c) The number of Indians successfully repatriated from conflict zones after government intervention.
(d) The decrease in the number of new recruitment cases reported after government actions.

Head Office: 127, Zone II, MP Nagar, Bhopal |+91-7676564400| https://www.toprankers.com Page 32 of 36
105. Which piece of evidence, if true, would most strengthen the claim that the Indian government's advisories and
crackdown on recruiters will significantly reduce the number of Indians being duped into dangerous jobs in
Russia?
(a) A significant increase in the public's awareness of the dangers of such recruitments.
(b) A documented decrease in the number of visa applications for Russia following the crackdown.
(c) Testimonies from individuals who decided against going to Russia because of the advisories.
(d) A recent survey indicates increased awareness of dangers among citizens applying for visas for jobs.
106. Which of the following is likely to be true basis of the passage?
(a) the government's late response is solely responsible for the plight of Indians caught in the Russia-Ukraine
conflict
(b) The Indian government has comprehensive data on the exact number of Indians currently working in conflict

m
zones in Russia.

o
(c) The Ministry of External Affairs had been issuing warnings about the risks of working in conflict zones well

c
before the deaths of the two Indian nationals.

monitoring of online job advertisements.

rs .
(d) The Indian government's initial lack of awareness of the recruitment issue was partly due to insufficient

e
107. What can be concluded about the role of online video advertisements in influencing Indians to take up military
jobs in Russia based on the passage?

k
(a) They have been the primary source of information for Indians about these opportunities.

n
(b) They misrepresented the nature of the jobs, promising safety away from frontlines.

a
(c) They targeted individuals specifically looking for opportunities in conflict zones.

r
(d) They were part of a larger scheme that included direct recruitment through agencies.

p
o
108. Which parttr-of the passage
5E7F3A0B9C7E
1Dprovides the strongest evidence that the Indian government's diplomatic relations with
Moscow have not been effectively leveraged to secure the safety and return of Indians from the conflict zones?

T
(a) The MEA issuing advisories against being swayed by offers for military jobs in Russia.
(b) The government and India’s Embassy in Moscow working with the Russian Army for early discharges.
(c) The CBI raiding visa recruiters across India for duping people into dangerous jobs.
(d) The government's failure to explain the lack of forceful case-making with Russian authorities.

7U1M
tr-5Q7P3O0S9L

7U1M
tr-5Q7P3O0S9L

Head Office: 127, Zone II, MP Nagar, Bhopal |+91-7676564400| https://www.toprankers.com Page 33 of 36
SECTION - E : QUANTITATIVE TECHNIQUES

Directions (Q.109-Q.114): Study the following information carefully and answer the questions given below.
Below data is given regarding the number of sculptures made by four sculptors Ravi, Sanjay, Manoj, and Rajesh
in two weeks (week 1, and week 2). The number of sculptures made by Ravi, and Sanjay in week 1 are equal.
Total number of sculptures made by all four sculptors in two weeks is 860 and number of sculptures made in all
two weeks by Ravi, Sanjay, and Manoj are 132, 154 and 210, respectively. Number of sculptures made in week
1 are 11/20 times of the total number of sculptures made in 2 weeks by all four sculptors. Ratio of number of
sculptures made in week 1, and week 2, by Ravi and Rajesh are 2:5, and 1:3, respectively.
109. Given that number of sculptures made in week 1 by Ravi and Sanjay are equal, how many sculptures did each
of them make in week 1?
(a) 44 (b) 64 (c) 68 (d) 77
110. What is the total number of sculptures made by Rajesh in the two weeks combined?

c o m
(a) 258 (b) 364 (c) 304 (d) 220

rs .
111. Which of the following statements is correct regarding the number of sculptures made by Sanjay and Rajesh in

e
each week?
(a) Sanjay made 44 sculptures in week 2. (b) Rajesh made 132 sculptures in week 1.
(c) Sanjay made 88 sculptures in week 1.
112. How many sculptures did Sanjay make in week 2?

an k
(d) Rajesh made 204 sculptures in week 2.

r
(a) 68 (b) 90 (c) 25 (d) 204

made by Manoj
(a) 95
in0B
tr-5E7F3A 9C7E1D
week 1?
(b) 33 p
113. What is the difference between the number of sculptures made by Sanjay in week 2 and the number of sculptures

o (c) 44 (d) 55

1?
(a) 48 (b) 58
T
114. If Rajesh made 3 times as many sculptures in week 2 as in week 1, how many sculptures did he make in week

(c) 68 (d) 77
Directions (Q.115-Q.120): Study the following information carefully and answer the questions given below.
Monthly incomes of U, V, W, X, Y, and Z in a company are given. In August, each of them invested some part
of their income at some rate of either simple or compound interest and saved the remaining amount. The average
1M
7P3O0S9L7UMonthly
of their monthly incomes taken together istr-5Q Rs.32000. savings of W and V are in the ratio 4: 1
respectively and V invested Rs.2500 more than the amount saved by him. X, who invested 40% of his monthly
income at M% rate of simple interest for 5 years, got Rs. 3000 more interest than W. W invested 1/3rd of his
income and saved the remaining Rs.24000. Monthly incomes of Y and Z are in the ratio 2: 3 respectively. Y,
whose monthly income is Rs.5000 more than X, saves 30% of his income. Y and U invested at 10% and N%
rates of compound interest for 3 years and 2 years respectively. Monthly savings of Y and U taken together is
tr-Rs.15000. Z 1M
5Q7P3O0S9L7U invested 2/3 part of his monthly income at O% rate of simple interest for 5 years and received
Rs.15000 as interest. V invested at 10% rate of simple interest for 6 years and received interest Rs.2100 more
than the interest received by W. X invested Rs. 1500 more than V. Total interest received by X and U together
is Rs.21620.
115. What is the interest rate (M%) for X?
(a) 10% (b) 15% (c) 20% (d) 12%

Head Office: 127, Zone II, MP Nagar, Bhopal |+91-7676564400| https://www.toprankers.com Page 34 of 36
116. What is the monthly income of Z if the monthly incomes of Y and Z are in the ratio 2:3?
(a) Rs. 40000 (b) Rs. 45000 (c) Rs. 50000 (d) Rs. 55000
117. What is the compound interest rate (N%) for U?
(a) 10% (b) 15% (c) 20% (d) 12%
118. Given that V invested at 10% simple interest for 6 years and received Rs. 2100 more interest than W. What
amount did V invest?
(a) Rs. 7500 (b) Rs. 8500 (c) Rs. 9500 (d) Rs. 10500
119. What is the average monthly income of U, V, W and X?
(a) Rs. 31500 (b) Rs. 32000 (c) Rs. 29250 (d) Rs. 29750

m
120. What are the combined monthly savings of X and Z?

o
(a) Rs. 36000 (b) Rs. 33000 (c) Rs. 39000 (d) Rs. 30000

rs .c
k e
r an
tr-5E7F3A0B9C

o
7E 1D

p
T
7U1M
tr-5Q7P3O0S9L

Mock Objection Form (MOF)

7U1M
tr-5Q7P3O0S9L

Head Office: 127, Zone II, MP Nagar, Bhopal |+91-7676564400| https://www.toprankers.com Page 35 of 36
Notes:

c o m
rs .
k e
r an
tr-5E7F3A0B9C

o
7E 1D

p
T
7U1M
tr-5Q7P3O0S9L

7U1M
tr-5Q7P3O0S9L

Head Office: 127, Zone II, MP Nagar, Bhopal |+91-7676564400| https://www.toprankers.com Page 36 of 36

You might also like